You are on page 1of 139

Higher School of Nanoscience and Nanotechnology

First Year Preparatory Classes

Support for Courses and Tutorials in


Mathematical Analysis

Slimane Azoug

2023 — 2024
(ESNN)S.Azoug

2
Contents

0.1 Terms to Learn . . . . . . . . . . . . . . . . . . . . . . . . . . . . . . . . . . 8

1 Real Numbers 9
1.1 Sets . . . . . . . . . . . . . . . . . . . . . . . . . . . . . . . . . . . . . . . . 9
1.1.1 Operations in R . . . . . . . . . . . . . . . . . . . . . . . . . . . . . . 9
1.1.2 The Fourteen Axioms in R . . . . . . . . . . . . . . . . . . . . . . . . 9
1.1.3 Notable Identities . . . . . . . . . . . . . . . . . . . . . . . . . . . . . 10
1.1.4 Square Root . . . . . . . . . . . . . . . . . . . . . . . . . . . . . . . . 11
1.1.5 Absolute Value . . . . . . . . . . . . . . . . . . . . . . . . . . . . . . 12
1.1.6 Powers . . . . . . . . . . . . . . . . . . . . . . . . . . . . . . . . . . 14
1.1.7 Upper Bound . . . . . . . . . . . . . . . . . . . . . . . . . . . . . . . 14
1.1.8 Lower Bound . . . . . . . . . . . . . . . . . . . . . . . . . . . . . . . 15
1.1.9 Characterization of the Lower Bound . . . . . . . . . . . . . . . . . . 16
1.1.10 Floor Function . . . . . . . . . . . . . . . . . . . . . . . . . . . . . . 16
1.2 Series of Exercises on Real Numbers . . . . . . . . . . . . . . . . . . . . . . . 17

2 Number Sequences 21
2.0.1 Definition . . . . . . . . . . . . . . . . . . . . . . . . . . . . . . . . . 21
2.0.2 Monotonicity of a Sequence . . . . . . . . . . . . . . . . . . . . . . . 21
2.0.3 Bounded Sequence . . . . . . . . . . . . . . . . . . . . . . . . . . . . 22
2.0.4 Limit of a Sequence . . . . . . . . . . . . . . . . . . . . . . . . . . . 22
2.1 Series of exercises on sequence of real number . . . . . . . . . . . . . . . . . 27

3
(ESNN)S.Azoug CONTENTS
3 Real Functions of a Real Variable 29
3.1 Generalities . . . . . . . . . . . . . . . . . . . . . . . . . . . . . . . . . . . . 29
3.1.1 monotonicity . . . . . . . . . . . . . . . . . . . . . . . . . . . . . . . 29
3.1.2 Algebraic operations . . . . . . . . . . . . . . . . . . . . . . . . . . . 30
3.1.3 Bounded Functions . . . . . . . . . . . . . . . . . . . . . . . . . . . . 31
3.1.4 The parity of a function . . . . . . . . . . . . . . . . . . . . . . . . . 31
3.1.5 Periodic Functions . . . . . . . . . . . . . . . . . . . . . . . . . . . . 32
3.2 Limits . . . . . . . . . . . . . . . . . . . . . . . . . . . . . . . . . . . . . . . 32
3.2.1 Operations on Limits . . . . . . . . . . . . . . . . . . . . . . . . . . . 34
3.2.2 Theorems on Limits . . . . . . . . . . . . . . . . . . . . . . . . . . . 34
3.2.3 Calculating Limits . . . . . . . . . . . . . . . . . . . . . . . . . . . . 35
3.3 Continuity . . . . . . . . . . . . . . . . . . . . . . . . . . . . . . . . . . . . . 35
3.3.1 Definitions . . . . . . . . . . . . . . . . . . . . . . . . . . . . . . . . 35
3.3.2 Continuity Extension . . . . . . . . . . . . . . . . . . . . . . . . . . . 35
3.3.3 Global Continuity . . . . . . . . . . . . . . . . . . . . . . . . . . . . 36
3.3.4 Operations on Continuous Functions . . . . . . . . . . . . . . . . . . . 36
3.3.5 Theorems on Continuous Functions . . . . . . . . . . . . . . . . . . . 36
3.4 The derivative of a function . . . . . . . . . . . . . . . . . . . . . . . . . . . . 38
3.4.1 The derivative and the continuity of a function . . . . . . . . . . . . . 38
3.4.2 Operations on the derivative function . . . . . . . . . . . . . . . . . . 39
3.4.3 Rolle’s Theorem . . . . . . . . . . . . . . . . . . . . . . . . . . . . . 40
3.4.4 Mean Value Theorem or Finite Increments Theorem . . . . . . . . . . 41
3.4.5 Generalized mean value theorem . . . . . . . . . . . . . . . . . . . . . 42
3.4.6 Optimality and Applications . . . . . . . . . . . . . . . . . . . . . . . 43
3.5 Series of exercises on functions . . . . . . . . . . . . . . . . . . . . . . . . . . 45

4 Common functions and their inverse 51


4.1 Inverse Function . . . . . . . . . . . . . . . . . . . . . . . . . . . . . . . . . . 51
4.1.1 Relation between function and the inverse . . . . . . . . . . . . . . . . 51
4.1.2 The derivative of the inverse function . . . . . . . . . . . . . . . . . . 52
4.2 Common Functions . . . . . . . . . . . . . . . . . . . . . . . . . . . . . . . . 52
4.2.1 Trigonometric Functions . . . . . . . . . . . . . . . . . . . . . . . . . 52
4.2.2 x → cos(x) . . . . . . . . . . . . . . . . . . . . . . . . . . . . . . . . 52
4.2.3 x → cos−1 (x) = arccos(x) . . . . . . . . . . . . . . . . . . . . . . . . . 53
4.2.4 x → sin(x) . . . . . . . . . . . . . . . . . . . . . . . . . . . . . . . . 54

4
(ESNN)S.Azoug CONTENTS
4.2.5 x → sin−1 (x) . . . . . . . . . . . . . . . . . . . . . . . . . . . . . . . 54
4.2.6 x → tan(x) . . . . . . . . . . . . . . . . . . . . . . . . . . . . . . . . 55
4.2.7 x → arctan(x) . . . . . . . . . . . . . . . . . . . . . . . . . . . . . . . 55
4.3 Hyperbolic Functions . . . . . . . . . . . . . . . . . . . . . . . . . . . . . . . 56
4.3.1 x → cosh(x) . . . . . . . . . . . . . . . . . . . . . . . . . . . . . . . 56
4.3.2 x → cosh−1 (x) = arg cosh(x) . . . . . . . . . . . . . . . . . . . . . . . 56
4.3.3 x → sinh(x) . . . . . . . . . . . . . . . . . . . . . . . . . . . . . . . . 57
4.3.4 x → sinh−1 (x) = arg sinh(x) . . . . . . . . . . . . . . . . . . . . . . . 57
e x −e−x
4.3.5 x → tanh(x) = sinh(x)
cosh(x)
= e x +e−x
. . . . . . . . . . . . . . . . . . . . . . . 57
4.3.6 Important relationships and identities . . . . . . . . . . . . . . . . . . 58
4.4 Series of exercises on common functions and inverse . . . . . . . . . . . . . . 59

5 limit development 61
5.1 Comparison of Functions in the Vicinity of a Point. Landau Notations . . . . . 61
5.1.1 Definitions and Properties . . . . . . . . . . . . . . . . . . . . . . . . 61
5.1.2 Properties of Equivalence . . . . . . . . . . . . . . . . . . . . . . . . 63
5.2 Taylor Polynomials . . . . . . . . . . . . . . . . . . . . . . . . . . . . . . . . 63
5.2.1 Maclaurin Series of Elementary Functions . . . . . . . . . . . . . . . . 65
5.2.2 Algebraic Operations on Taylor Series . . . . . . . . . . . . . . . . . . 68
5.2.3 Generalized Taylor Expansion . . . . . . . . . . . . . . . . . . . . . . 70
5.2.4 Applications of optimization calculations for functions . . . . . . . . . 71
5.3 Series of exercises on Taylor expansions . . . . . . . . . . . . . . . . . . . . . 73
5.3.1 Corrections . . . . . . . . . . . . . . . . . . . . . . . . . . . . . . . . 76

6 Integrals 79
6.1 Antiderivatives functions . . . . . . . . . . . . . . . . . . . . . . . . . . . . . 79
6.2 Properties . . . . . . . . . . . . . . . . . . . . . . . . . . . . . . . . . . . . . 80
6.3 Computation of antiderivatives . . . . . . . . . . . . . . . . . . . . . . . . . . 80
6.4 Definite Integrals . . . . . . . . . . . . . . . . . . . . . . . . . . . . . . . . . 81
6.5 Series of exercises on integrals . . . . . . . . . . . . . . . . . . . . . . . . . . 83

5
(ESNN)S.Azoug CONTENTS
7 Differential Equations 85
7.1 Introduction . . . . . . . . . . . . . . . . . . . . . . . . . . . . . . . . . . . . 85
7.2 First-order Differential Equations . . . . . . . . . . . . . . . . . . . . . . . . . 87
7.2.1 Separable Differential Equations . . . . . . . . . . . . . . . . . . . . . 87
7.2.2 First-Order Homogeneous Differential Equations . . . . . . . . . . . . 88
7.2.3 First-Order Differential Equations . . . . . . . . . . . . . . . . . . . . 88
7.2.4 Bernoulli’s Differential Equation . . . . . . . . . . . . . . . . . . . . . 90
7.2.5 Riccati’s Differential Equation . . . . . . . . . . . . . . . . . . . . . . 90
7.3 Second-Order Homogeneous Differential Equations with Constant Coefficients 91
7.4 Second-Order Non-Homogeneous Differential Equations with Constant Coef-
ficients . . . . . . . . . . . . . . . . . . . . . . . . . . . . . . . . . . . . . . . 93
7.5 Série d’exercices sur les équations différentielles . . . . . . . . . . . . . . . . 94
7.5.1 Correction des exercices sur les équations différentielles . . . . . . . . 96

8 Functions of Two Variables 97


8.1 Generalities . . . . . . . . . . . . . . . . . . . . . . . . . . . . . . . . . . . . 97
8.1.1 Topology Vocabulary in R2 . . . . . . . . . . . . . . . . . . . . . . . . 97
8.2 Functions with Two Variables . . . . . . . . . . . . . . . . . . . . . . . . . . . 98
8.2.1 Partial Functions in R2 . . . . . . . . . . . . . . . . . . . . . . . . . . 98
8.2.2 Representation by Level Curves . . . . . . . . . . . . . . . . . . . . . 98
8.2.3 Limits and Continuity . . . . . . . . . . . . . . . . . . . . . . . . . . 99
8.3 Practical Computation of Limits . . . . . . . . . . . . . . . . . . . . . . . . . 100
8.3.1 The Function Has a Limit at M0 . . . . . . . . . . . . . . . . . . . . . 100
8.3.2 The Function Does Not Have a Limit at M0 . . . . . . . . . . . . . . . 100
8.4 Differential Calculus . . . . . . . . . . . . . . . . . . . . . . . . . . . . . . . 101
8.4.1 Functions of Class C 1 . . . . . . . . . . . . . . . . . . . . . . . . . . 101
8.4.2 Differentiability . . . . . . . . . . . . . . . . . . . . . . . . . . . . . . 101
8.4.3 First Order Taylor Expansion . . . . . . . . . . . . . . . . . . . . . . . 102
8.4.4 Tangent Plane to a Surface . . . . . . . . . . . . . . . . . . . . . . . . 102
8.4.5 Gradient . . . . . . . . . . . . . . . . . . . . . . . . . . . . . . . . . . 102
8.4.6 Jacobian Matrix . . . . . . . . . . . . . . . . . . . . . . . . . . . . . . 103
8.5 Second Partial Derivatives . . . . . . . . . . . . . . . . . . . . . . . . . . . . 104
8.6 Extrema . . . . . . . . . . . . . . . . . . . . . . . . . . . . . . . . . . . . . . 104
8.7 Lagrange Multipliers . . . . . . . . . . . . . . . . . . . . . . . . . . . . . . . 106
8.8 Cobb-Douglas Function . . . . . . . . . . . . . . . . . . . . . . . . . . . . . . 106

6
(ESNN)S.Azoug CONTENTS
8.9 Series of exercises on functions of two variables . . . . . . . . . . . . . . . . . 109
8.9.1 Correction of the series of exercises on functions of two variables . . . 112

9 Courbes Planes 119

10 Exercise Solutions 121


10.1 Solutions to exercises on real numbers . . . . . . . . . . . . . . . . . . . . . . 121
10.2 Solutions to exercises on functions . . . . . . . . . . . . . . . . . . . . . . . . 127
10.3 Solutions to exercises on integrals . . . . . . . . . . . . . . . . . . . . . . . . 136
10.3.1 Fixed Point Theorem . . . . . . . . . . . . . . . . . . . . . . . . . . . 138
10.3.2 Result on Fixed Point . . . . . . . . . . . . . . . . . . . . . . . . . . . 138

7
(ESNN)S.Azoug CONTENTS
0.1 Terms to Learn
Real Numbers: Natural number, integer, rational number, irrational number, real numbers,
variable x, y, belongs to, equation, inequality, empty set, complex numbers, real, imaginary,
modulus, argument, conjugate, inverse, root, complex numbers, empty set, complementary, in-
clusion, belongs to, absolute value, floor, root, interval, subset.

Numerical Sequences: Sequences, arithmetic, geometric, common difference, sum, general


term, convergent, divergent, increasing, decreasing, constant, alternating, even, odd, periodic,
solution, lower, upper, first-order equation, second-order equation, derivative, inflection point,
reasoning by induction, maximum, minimum, bounded, upper bounded, lower bounded, mono-
tonicity, discriminant, open, closed, semi-open, indeterminate form.

Functions: Domain of definition, limit, function, mapping, continuity, differentiability, asymp-


tote (horizontal, vertical, oblique), graph, curve, variation table, monomial, polynomial, inter-
val, exponential, logarithm, tangent, slope, symmetry center, axis of symmetry, x-axis, y-axis,
orthonormal, half-line, affine, linear, parabola, extension by continuity, differentiable, continu-
ous, inverse of a function, orthonormal coordinate system.

Integrals: Primitive, bounds, integral, area.

8
Chapter 1
Real Numbers

1.1 Sets

Figure 1.1: Sets

1.1.1 Operations in R
We define addition as: (x, y) 7→ x + y
We define multiplication as: (x, y) 7→ x.y
The set of real numbers is a commutative field for addition and multiplication. We define a
relation of order as follows:

(x, y) 7−→ x < y

1.1.2 The Fourteen Axioms in R


A1: ∀x, y ∈ R, x + y = y + x
A2: ∀x, y, z ∈ R, (x + y) + z = x + (y + z)

9
(ESNN)S.Azoug CHAPTER 1. REAL NUMBERS
A3: ∀x ∈ R, x + 0 = 0 + x = x
A4: ∀x ∈ R, ∃(−x) ∈ R, x + (−x) = 0.
A5: ∀x, y ∈ R, x · y = y · x
A6: ∀x, y, z ∈ R, (x · y) · z = x · (y · z)
A7: ∀x ∈ R, x · 1 = 1 · x = x
A8: ∀x ∈ R∗ , ∃(x−1 ) ∈ R, such that x · (x−1 ) = 1.
A9: ∀x, y, z ∈ R, x · (y + z) = x · y + x · z
A10: ∀x, y, z ∈ R, one and only one of the following holds: x = y, x < y, x > y.
A11: ∀x, y, z ∈ R, if x > y and y > z, then x > z.
A12: ∀x, y, z ∈ R, if x < y, then x + z < y + z.
A13: ∀x, y, z ∈ R, if (z > 0) and x < y, then x · z < y · z.
A14: Axiom of the supremum.

1.1.3 Notable Identities


Pascal’s Triangle
Newton’s Binomial Formula

Figure 1.2: Pascal’s Triangle

(a + b)n =
Pn r n−r
r=0 C n a · br

an − bn = (a − b) · (an−1 + an−2 · b + ... + bn−1 )

Proof by Induction.

P(n) : an − bn = (a − b) · (an−1 + an−2 · b + ... + bn−1 )

S = (an−1 + an−2 · b + ... + bn−1 )

10
(ESNN)S.Azoug 1.1. SETS
P(n) : an − bn = (a − b) · S

an+1 = an · a = an · (a − b + b) = an · (a − b) + b · an

−bn+1 = −bn · b = −bn · (b − a + a) = −bn · (b − a) − a · bn

2(an+1 − bn+1 ) = (a − b) · (an + bn ) + (a + b) · (an − bn )


Conclude.
For any given real numbers a and b,

• (a + b)2 = 1 · a2 + 2 · ab + 1 · b2

• (a + b)3 = 1 · a3 + 3 · a2 b + 3 · ab2 + 1 · b3

• (a + b)4 = 1 · a4 + 4 · a3 b + 6 · a2 · b2 + 4 · ab3 + 1 · b4

• (a + b)5 = 1 · a5 + 5 · a4 b1 + 10 · a3 · b2 + 10 · a2 · b3 + 5 · a1 · b4 + 1 · b5

• a2 − b2 = (a − b) · (a + b)

• a3 − b3 = (a − b) · (a2 + ab + b2 )

• a4 − b4 = (a − b) · (a3 + a2 b + ab2 + b3 )

• a5 − b5 = (a − b) · (a4 + a3 b + a2 · b2 + ab3 + b4 )

1.1.4 Square Root

The square root of a real number a (> 0) is defined by the two solutions of the following
equation:
x2 = a
√ √
These two solutions are x = a and x = − a.
Let’s recall the essential properties of square roots.

11
(ESNN)S.Azoug CHAPTER 1. REAL NUMBERS
Properties For all positive numbers x and y, we have:

√ √ √
• x.y = x. y
q √
• x
y
= √
x
y


• x2 =| x |
√ √ √
• x+y6 x+ y

Applications

• Show that:
Cn0 + Cn1 + ..Cnn = 2n
Cn0 − Cn1 + ..(−1)nCnn = 0.

• Expand: (x + 1)5

• Deduce: (x − 1)5

Solutions

• (1 + 1)n = Cn0 + Cn1 + ..Cnn


(1 + (−1))n = Cn0 − Cn1 + ..(−1)nCnn = 0.

• Expand: (x + 1)5 = x5 + 5x4 + 10x3 + 10x2 + 5x + 1

• Deduce: (x + (−1))5 = x5 − 5x4 + 10x3 − 10x2 + 5x − 1

1.1.5 Absolute Value


The absolute value of a real number x, denoted as | x |, is defined as follows:

x

 if x ≥ 0
| x |= 
−x if x < 0

Alternatively, | x |= max{x, −x}.

12
(ESNN)S.Azoug 1.1. SETS
• | x · y |=| x | · | y |

• x
y
= |x|
|y|

• | x + y |≤| x | + | y | (Triangle inequality).

• || x | − | y ||≤| x − y |

Proof
| x + y |6| x | + | y |

• (| x | + | y |)2 − (| x + y |)2 =| x |2 + | y |2 +2 | x | · | y | −(x + y)2

• (| x | + | y |)2 − (| x + y |)2 = 2(| x · y | −xy) > 0.


− | x |6 x 6| x |
− | xy |6 xy 6| xy |

• (| x | + | y |)2 − (| x + y |)2 > 0.

• | x | + | y |>| x + y | Q.E.D.

• | x |= |x − y + y| ≤| x − y | + | y |
• =⇒| x | − | y |≤| x − y |
• Similarly, to get | y | − | x |≤| x − y |
• Deduce: − | x − y |≤| x | − | y |≤| x − y |
• || x | − | y ||≤| x − y | Q.E.D.
• (| x | + | y |)2 − (| x + y |)2 = 2(| x · y | −xy)
• − | x · y |≤ xy ≤| x · y |=⇒| x · y | −xy ≥ 0 Q.E.D.

• | x |=| x − y + y |) ≤| x − y | + | y |
• =⇒| x | − | y |≤| x − y |
• with the same manner : | y | − | x |≤| x − y |
• deduce that : − | x − y |≤| x | − | y |≤| x − y |
• || x | − | y ||≤| x − y | CQFD.

13
(ESNN)S.Azoug CHAPTER 1. REAL NUMBERS
1.1.6 Powers

1.1.7 Upper Bound

For a non-empty subset A of R, we say that A is bounded above if there exists a real num-
ber M such that:

∀x ∈ A : x 6 M .

M is called the upper bound of A.

Application Determine the upper bounds and lower bounds of the following sets:

• A1 = {0, 1, 3, 4}

• A2 = N∗

• A3 =] − 3, 4] ∪ [5, 10]

Solutions The upper bounds and lower bounds of the following sets:

• A1 = {0, 1, 3, 4}
Upper bounds = {4}, Lower bounds = {0}.

• A2 = N∗
Upper bounds = empty set, Lower bounds = {0}.

• A3 =] − 3, 4] ∪ [5, 10]
Upper bounds = [10, +∞ [, Lower bounds = −∞, −3].

Axiom of the Supremum

Every non-empty subset of R that is bounded above has a supremum.

Characterization of the Supremum

(M = sup(A)) ⇐⇒ (∀x ∈ A : x 6 M) and (∀ε > 0, ∃x∗ ∈ A/M − ε < x∗ 6 M)


Another equivalent characterization is used:
(M = sup(A)) ⇐⇒ (∀x ∈ A : x 6 M) and (there exists a sequence of elements Un ∈ A such that lim Un = M)

14
(ESNN)S.Azoug 1.1. SETS
Application Let
2n2
A={ /n ∈ N}
n2 + 1
Show that sup A = 2.

Hint
2
• Calculate: lim n2n
2 +1

2n2
• Show that: n2 +1
6 2.

1.1.8 Lower Bound


For a non-empty subset A of R, we say that A is bounded below if there exists a real num-
ber m such that:

∀x ∈ A : x > M .

m is called the lower bound of A.


The greatest of the lower bounds is called the infimum of A.

Application Soit
4
B = {3 + /n ∈ N∗ }
n
Determiner Sup (B), Inf(B), Max(B), Min(B).

Indication Montrer que : 3 < 3 + 4


n
67

Solution

• Sup (B)=7, Inf(B)=3.

• Max(B)=7, Min(B) n’existe pas.

15
(ESNN)S.Azoug CHAPTER 1. REAL NUMBERS
1.1.9 Characterization of the Lower Bound
: (m = inf(A)) ⇐⇒ (∀ε > 0, ∃x∗ ∈ A/m 6 x < ε + m)

1.1.10 Floor Function


Definition 1.1.1. For any real number x, there exists an integer denoted as E(x) such that:

E(x) 6 x < E(x) + 1 Sometimes denoted as [x].

• x − 1 < E(x) 6 x

• ∀p ∈ Z, E(x + p) = E(x) + p

• x −→ E(x) (is an increasing function).

16
(ESNN)S.Azoug 1.2. SERIES OF EXERCISES ON REAL NUMBERS
1.2 Series of Exercises on Real Numbers
Exercise: 01.(Knowledge Check)
Determine whether the following statements are true or false:
Let a, b, and k be real numbers such that 0 6 a 6 b. In this case, we have:

1. a · k 6 b · k

2. a · k 6 a · k2

3. a · k 6 a · k3
a b
4. 6
k k
a b
5. 4
6 4
k k
Exercise: 02.
Determine whether the following statements are true or false:
Let a, b, c, and k be real numbers such that 0 6 a 6 b 6 c. In this case, we have:

1. a · k 6 b · k 6 c · k

2. a · k 6 b · k2 6 c · k3

3. a · k2 6 b · k2 6 c · k2
a b c
4. 6 6
k k k
a b c
5. 2
6 2 6 2
k k k
Exercise: 03.(Applications)
Show that for all a, b, c in R the following hold:

a b
1. + >2
b a
2ab √ a+b
2. 6 ab 6
a+b 2
3. Deduce: ab + bc + ac 6 a2 + b2 + c2

Exercise: 04.(Applications)
Let x, y be in R such that:
1 6 x 6 2, and 2 6 y 6 3.
Provide an estimate for A, B:
!2 s
2x + 1 2x2 + 2x + 1
A= , B=
3y − 1 x3 + 3y − 1

17
(ESNN)S.Azoug CHAPTER 1. REAL NUMBERS
Exercise: 05.(Applications)
Let x, y be in R, solve the following systems of equations:

2x − 5 < 4x − 2




6x + 1

>1




2

2x + 8y = −6



 x − y2 = 1

O5 = x, O6 = 6 · |y|
Exercise: 06.(Knowledge Check)
Determine whether the following statements are true or false:
Let a, b, c, and k be real numbers:

1. 2 x + 2 x = 22x

2. (an + bn )2 = an + bn + 2 · an · bn

Exercise: 07.(Applications)
Simplify the following expression:

a x .xy .x2
O3 = with x = y + 2
ay .a2 .x x

Exercise: 08. (Knowledge Check)


Let x, y be real numbers.

1. Expand (x + y)4 and (x − y)5

2. Find the coefficients of x15 in the expansion of (x + 2)20

O1 = (coefficient of x2 ) + 2 in the expansion of (x + 1)4


Exercise: 09. (Knowledge Check)
Determine if the following statements are true or false: Let a, b be real numbers, then:

1. a2 = a

2. ( a)2 = a
√ √ √
3. a· b= a·b
√ √ √
4. a·b= a· b

5. a2 + b2 = a + b
√ √ √
6. a+b= a+ b

(a + b)2 = a + b
p
7.

18
(ESNN)S.Azoug 1.2. SERIES OF EXERCISES ON REAL NUMBERS
Exercise: 10. (Applications)
Show that for all positive real numbers a, b, c:
√ √ √
1. a+b6 a+ b

2. ab + bc + ac 6 a + b + c

Exercise: 11. (Applications)


Solve in R:
√ √
1. 2x − 1 6 4x + 1

2. x − 2 6 3x + 1

3. x − 4 · 4x − 19 > 1

Exercise: 12. (Knowledge Check)


Determine if the following statements are true or false: Let a, b be real numbers:

1. | a2 · b | = b · a2

2. | a2 | = a2

3. | a + b | = | a | + | b |

4. | a − b | 6 | a | + | b |

Exercise: 13. (Applications)


Solve in R:

1. | 2x + 1 | = x − 1, (O2 = 5x + 1)

2. | 2x − 1 | 6 3

3. | 2x − 2 | + | x + 1 | = 3

4. | 2x − 2 | − | x + 1 | 6 3

Exercise: 14.
Find, if they exist, the maximum, minimum, supremum, and infimum of the following sets:

1. A = {−1, 7, 8, −3, 13}, (O4 = sup(A))


( )
1
2. B = 1 − , n ∈ N ∗
n
( )
1 n
3. C = + ,n ∈ N
2 1+n

19
(ESNN)S.Azoug CHAPTER 1. REAL NUMBERS
Exercise : 15. (Applications)
Let A, B be two non-empty subsets of R such that A ∩ B , ∅. If A ⊂ B, then:

⇒ sup(A) 6 sup(B), inf(A) > inf(B)

Exercise : (Code to find?)


Find the following word:
Pro f : O1 .O2 O3 O4 O5 O6

20
Chapter 2
Number Sequences

2.0.1 Definition
Definition 2.0.1. A number sequence is a mapping from D ⊆ N −→ R that assigns to every
natural number n its real number image U(n), denoted as Un .

Remarks There are two methods to define a sequence:


2n + 2
1) Un = , a sequence defined by its general term Un as a function of n.
n−1
2) Un+1 = Un + Un−1 , a sequence defined by a recursive relation and initial values U0 , U1 .

2.0.2 Monotonicity of a Sequence


• A sequence Un is said to be increasing (resp. strictly increasing) if
∀n ∈ N, Un 6 Un+1 (resp.Un < Un+1 )

• A sequence Un is said to be decreasing (resp. strictly decreasing) if


∀n ∈ N, Un > Un+1 (resp.Un > Un+1 )

• A sequence Un is said to be constant if


∀n ∈ N, Un = Un+1

Application Study the monotonicity of the following sequences.


2n Un+1 2
1) Un = , (Hint: show = , compare with 1), conclude.
n! Un n+1
2n + 4
2) Un = , (Hint: Calculate Un+1 − Un , compare with 0).
5n − 2

21
(ESNN)S.Azoug CHAPTER 2. NUMBER SEQUENCES
Solution

Un+1 2
• = <1
Un n+1
implies: Un+1 < Un , decreasing.
−24
• Un+1 − Un = <0
25n + 5n − 6
2
implies: Un+1 < Un , decreasing.

2.0.3 Bounded Sequence


• A sequence Un is said to be bounded above if

∀n ∈ N, ∃M ∈ R Un 6 M

• A sequence Un is said to be bounded below (resp. strictly bounded below) if

∀n ∈ N, ∃m ∈ R Un > m

• A sequence Un is said to be bounded if

∀n ∈ N, ∃m, M ∈ R m 6 Un 6 M

cos(n)
Application. Prove that the sequence defined by Un = 2 is bounded.
n +1
(Hint: −1 6 cos(n) 6 1).

2.0.4 Limit of a Sequence

(lim Un = l) ⇐⇒ (∀ε > 0, ∃N, n > N ⇒| Un − l |< ε)


(lim Un = +∞) ⇐⇒ (∀M ∈ R, ∃N, n > N ⇒ Un > M

Application. Using the definition, prove that the limit of the sequence defined by
1
Un = 1 + is equal to 1.
n+1
Theorem 2.0.1. (Uniqueness)
The limit of a convergent sequence is unique.

**Proof:** By contradiction, assume :


ε ε
ε = |l1 − l2 | 6 |Un − l1 | + |Un − l2 | < + ⇒ε<ε
2 2
22
(ESNN)S.Azoug
Properties 2.0.1. Let (Un ), (Vn ) be two convergent sequences. Then, the sequences (Un + Vn ),
Un
(α.Un ), |Un |, (Un − Vn ), are convergent, and we have:
Vn
- lim(Un + Vn ) = lim Un + lim Vn = l1 + l2 .
- lim(α.Un ) = α. lim(Un ) = α.l1
- lim(|Un |) = | lim Un | = |l1 |
- lim(Un .Vn ) = lim Un . lim Vn = l1 .l2 .
Un lim Un l1
- lim( ) = = , if l2 , 0.
Vn lim Vn l2

**Application:** Show that the limit of the sequence defined by :


1
lim n n = 1.
 a n
lim 1 + = ea .
n
Proposition 2.0.1. Every convergent sequence is bounded.

**Proof:** Write:
− | l | −ε < l − ε < Un < l + ε <| l | −ε = K
Conclude that K is the upper bound.

**Remarks:** 1. An unbounded sequence cannot be convergent.


2. A bounded sequence is not necessarily convergent (consider Un = (−1)n ).

Theorem 2.0.2. (Monotone Convergence Theorem)


- A sequence (Un ), n ∈ N that is increasing and bounded is convergent, and lim Un = sup Un .
- A sequence (Un ), n ∈ N that is increasing and unbounded diverges to +∞.
- A sequence (Un ), n ∈ N that is decreasing and bounded is convergent, and lim Un = inf Un .
- A sequence (Un ), n ∈ N that is decreasing and unbounded diverges to −∞.

Theorem 2.0.3. (Sandwitch)


Let (Un ), n ∈ N be a sequence. If there exist two convergent sequences (Vn ), n ∈ N and (Wn ),
n ∈ N both converging to the same limit l ∈ R such that:

∃N ∗ ∈ N, ∀n ∈ N, n > N ∗ ⇒ Vn 6 Un 6 Wn
**then lim Un = l.**

n
Application Show that :lim Un = lim (−1)
n
= 0.

Definition 2.0.2. (Subsequence) Let (Un ), n ∈ N, be a sequence. A subsequence or subsequence


of (Un ), n ∈ N, is any sequence (yn ) where yn = Uϕ(n) , and ϕ : N −→ N is a strictly increasing
function.

23
(ESNN)S.Azoug CHAPTER 2. NUMBER SEQUENCES
n
Application: Find two subsequences of the sequence defined by Un = (−1)
n
.
Solution: U2k = 2k1 . U2k+1 = 2k+1
−1
.

Definition 2.0.3. (Limit Point) A real number a ∈ R is a limit point of the sequence (Un ) if and
only if there exists a convergent subsequence that converges to a.

Theorem 2.0.4. • Let (Un ) be a sequence. If (Un ) converges to l, every subsequence of


(Un ) also converges to l.

• If a subsequence of (Un ) diverges, or if two subsequences of (Un ) have different limits,


then (Un ) diverges.

• If two subsequences of (Un ) converge to the same limit l, and if (Un ) is a term of one of
these subsequences, then (Un ) also converges to l.

Proposition 2.0.2. : If (U2n ) and (U2n+1 ) converge to l, then (Un ) converges to l.

Definition 2.0.4. (Adjacent Sequences) The sequences (Un ) and (Vn ) are adjacent if: - (Un ) is
increasing, - (Vn ) is decreasing, - lim(Un − Vn ) = 0.

Theorem 2.0.5. If two sequences are adjacent, they are convergent and have the same limit.

Arithmetic Sequence A sequence (Un ) is an arithmetic sequence with a common difference


r ∈ R and initial term U0 if:
∀n ∈ N, Un+1 = Un + r.
It can be observed that:
∀n ∈ N, Un+1 = Un + nr.

Geometric Sequence A sequence (Un ) is a geometric sequence with a common ratio r ∈ R and
initial term U0 if:
∀n ∈ N, Un+1 = r · Un .

Note:
∀n ∈ N, Un+1 = U0 · rn+1

Theorem 2.0.6. (Convergence of a Geometric Sequence)


Let (Un ) be a geometric sequence with a common ratio r ∈ R and initial term U0 .
- If r < −1, the sequence (Un ) is divergent and has no limit.
- If r = −1, the sequence (Un ) diverges and has two limit points: 1 and -1.
- If |r| < 1, the sequence (Un ) converges to 0.
- If r = 1, the sequence (Un ) is constant.
- If r > 1, the sequence (Un ) is divergent.

24
(ESNN)S.Azoug
Definition: Arithmetic-Geometric Sequence** A sequence (Un ) is an arithmetic-geometric
sequence with initial term U0 if:

∀n ∈ N, Un+1 = a · Un + b,

where a and b are real numbers.


- If a = 1, the sequence (Un ) is an arithmetic sequence with a common difference b.
- If b = 0, the sequence (Un ) is a geometric sequence with a common ratio a.
- If |a| < 1, the sequence (Un ) converges to 0.
n −1
- If a , 1, the sequence Un = U0 · an + aa−1 · b.

**Task**
**Questions:** Based on figures 2.1-2.2, to calculate the shaded area in the unit square start-
ing with s1 = 164 , show that it forms an arithmetic-geometric sequence. Then, calculate the
complete area when this operation is repeated infinitely.
Convergence of an Arithmetic-Geometric Sequence **A sequence (Un ) is an arithmetic-
geometric sequence with initial term U0 and converges if and only if |a| < 1, and its limit is
b
1−a
.

Definition 2.0.5. (Linear Second Order Recurrent Sequence) A linear second-order recurrent
sequence is written as follows:

U0 = α.




U1 = β.




U = a · U + b · U .


n+2 n+1 n

Writing the General Term of a Linear Second Order Recurrent Sequence To do this, we
write the characteristic equation:
r2 − a · r − b = 0.
We calculate

the discriminant:

∆ = a2 + 4b - If ∆ > 0, the equation has two distinct roots:
r1 = a−2 ∆ and r2 = a+2 ∆ , where α, β are real numbers.

Un = α · r1n + β · r2n .

- If ∆ = 0, the equation has a double root: r1 = r2 = a


2
 a 2
Un = (α + β · n) · .
2

- If ∆ < 0, the equation has two complex roots: Z = [l, θ], Z = [l, −θ]

Un = (l)n · (α · cos(n · θ) + β · sin(n · θ)).

**Application**

25
(ESNN)S.Azoug CHAPTER 2. NUMBER SEQUENCES
Consider the following sequence:

U0 = 1.




U1 = −1.




U = 3 · U − 2 · U .


n+2 n+1 n

Write the general term in terms of n.


**Solution:** Un = 3 − 2n+1

26
(ESNN)S.Azoug 2.1. SERIES OF EXERCISES ON SEQUENCE OF REAL NUMBER
2.1 Series of exercises on sequence of real number
n(n + 1)(2n + 1)
Exercise: 01.(Recurrence) Prove the following relations by induction: 1. nk=1 k2 =
P
6
1 n
2. 1 + 3 + 5 + .... + (2n − 1) = n 3. k=1 =
2 Pn
(2k − 1)(2k + 1) 2n + 1

Exercise: 02.(Sum and Product) Express using the symbols Π and up to order n the follow-
P
e1 e2 e3
1
ing relations: 1. 1·2 + 2·3
1
+ 3·4
1
... 2. 1·2·2
1
1 + 2·3·22 + 3·4·23 ... 3. sin(2x) + sin(3x2 ) + sin(5x3 ) ... 4. 1 · 2 · 3 ...
2 3 1 1 1

Exercise: 03.(Knowledge Check) 1. Is the sum of two divergent sequences necessarily diver-
gent? 2. Can the product of two divergent sequences result in a convergent sequence? 3. If
(Un ) is a decreasing positive sequence, is it necessarily true that lim Un = 0?
Exercise: 04.(Nature of a Sequence) √
Study the√nature of the following sequences: (a, α are real numbers) 1. a)Un = n − n2 − n,
n−1
b)Un = √
n+1
nn
2. c)Un = , d)Un = (1 + na )n
n!
cos(nα)
3. e)Un = , f )Un = 2n − 3n
n
Exercise: 05.(Bounding)
Consider the numerical sequence (Un ) defined by Un = nk=0 n+1√k . Using suitable bounding,
P
determine the limit of this sequence.
Exercise: 06.(Convergence Theorem)
U2
Consider the numerical sequence (Un ) defined by U0 = 1 and Un+1 = n + 1. 1. Prove by
4
induction that (Un ) is increasing and bounded by 2.
2. Calculate its limit.
Exercise: 07.
Let θ be a real number in the interval [0, π2 ].

Consider the sequence (Un ) defined by: U0 = 2 cos(θ), Un+1 = 2 + Un .
1. Compute U1 , U2 as functions of θ.
2. Show that Un = 2 cos( 2θn ) and find its limit.
Exercise: 08.
Show that the two sequences (Un ), (Vn ) defined by:
n
X 1 1 1
Un = , Vn = Un + − 2
k=3
1 + k2 n 2n
are adjacent.
Exercise: 09.
Let (Un ) be defined by:

U0 = 1, Un+1 = Un · e−Un
1. Show that this sequence is positive, decreasing, and converges. Calculate its limit.
2. Let S n = nk=1 Uk . Prove that for all n, Un+1 = e−S n .
P

27
(ESNN)S.Azoug CHAPTER 2. NUMBER SEQUENCES
3. Deduce the limit of S n .
Exercise: 10.(Fixed Point) Using the fixed-point theorem, prove that the recurrent sequence
defined by
1
Un+1 = · sin(Un )
2
converges.

Exercise: 11.(Equation of Leonardo de Pisa (1225))


Consider the function
f (x) = x3 + 2x2 + 10x − 20

1. Show that the equation f (x) = 0 has a real root in the interval ]1; 2[.
2. Prove that the equation f (x) = 0 can be written as F(x) = x, where
20
F(x) =
x2 + 2x + 10

3. Show that |F 0 (x)| < 1 for x in ]1; 2[.


4. Deduce that the sequence Un = F(Un−1 ) is convergent. What is its limit?
5. Taking U0 = 1, calculate U1 , U2 , ...U24 using a computer.

**Home-Work(+2 pts)**
1. **Leonardo de Pisa** found 1.368808107, now find U30 .
2. State the fixed-point theorem.
Exercise: 12. Consider the numerical sequence defined by:

Un+2 = −Un+1 + 2Un

with |α| < 1 and given U0 = 0, U1 = 3.


- Show Un = 1 − (−2)n .
P
- Express Un as a function of n.

Exercise: 13.
Consider the numerical sequence defined by:

Un+1 = α · Un + 1

with |α| < 1 and given U0 .


- Write Un as a function of n.
- Determine the limit of Un .
- Un = Vn + a, determine a so that (Vn ) is geometric.
P P
- Express Vn as a function of n, then find Vn .

28
Chapter 3
Real Functions of a Real Variable

3.1 Generalities
Definition 3.1.1. A function f is a relation that associates with every real number x in a set E
of R a real number y, denoted as y = f (x).
It is denoted as:
f : E −→ F
where D is the domain of the function f , i.e., the set of elements in E that have an image under
f.

**Equality of Two Functions:** Two functions f and g are equal if they have the same do-
main E, and if f (x) = g(x) for every x in E.

*Example:*
f : [0; 1] −→ R, f (x) = 2x + 5
g : [0; 3] −→ R, g(x) = 2x + 5
These two functions are not equal due to their different domains.

3.1.1 monotonicity
Definitions. Let f : E → R be a function defined on a subset E of R. We say that:
- f is increasing on E if for all x, y ∈ E, x ≤ y implies f (x) ≤ f (y);
- f is strictly increasing on E if for all x, y ∈ E, x < y implies f (x) < f (y);
- f is decreasing on E if for all x, y ∈ E, x ≤ y implies f (x) ≥ f (y);
- f is strictly decreasing on E if for all x, y ∈ E, x < y implies f (x) > f (y). Sure, let’s consider
a few examples to illustrate different types of monotonicity for functions: Examples
1. **Increasing Function:** f (x) = 2x + 3 This function is increasing because for any two
numbers x1 and x2 where x1 < x2 , f (x1 ) = 2x1 + 3 < 2x2 + 3 = f (x2 ).

29
(ESNN)S.Azoug CHAPTER 3. REAL FUNCTIONS OF A REAL VARIABLE
2. **Strictly Increasing Function:** f (x) = e x (where e is Euler’s number, approximately
2.71828) This function is strictly increasing because for any x1 < x2 , f (x1 ) = e x1 < e x2 = f (x2 ).

In all these examples, the functions exhibit different types of monotonicity based on the condi-
tions mentioned earlier.

3.1.2 Algebraic operations

Algebraic Operations on Functions:


Algebraic operations on functions involve mathematical manipulations such as addition, sub-
traction, multiplication, division, and composition. Here’s how these operations can be per-
formed on functions:
1. **Addition and Subtraction:** If f (x) and g(x) are two functions, then the sum ( f + g)(x)
and the difference ( f − g)(x) are defined as follows:

( f + g)(x) = f (x) + g(x)

( f − g)(x) = f (x) − g(x)

2. **Multiplication:** Multiplying two functions f (x) and g(x) results in a new function
( f × g)(x) defined by:
( f × g)(x) = f (x) × g(x)

3.
 f **Division:** Dividing two functions f (x) and g(x) (where g(x) , 0) gives a new function
g
(x) defined by:
!
f f (x)
(x) =
g g(x)
g(x) cannot be equal to zero, otherwise the division is undefined.
4. **Composition:** The composition of two functions f (x) and g(x) results in a new function
( f ◦ g)(x) defined by:
( f ◦ g)(x) = f (g(x))

Here, g(x) is evaluated first, and the result is then passed as an argument to the function f (x).
5. **Powers p and Roots:** If f (x) is a function, then ( f n )(x) represents f (x) raised to the
power n, and ( n f )(x) represents the nth root of f (x).

These algebraic operations on functions are used to create new functions from existing ones.

30
(ESNN)S.Azoug 3.1. GENERALITIES
3.1.3 Bounded Functions

A function f : E −→ F is said to be bounded on E if there exist real numbers M and m such


that:
∀x ∈ E, m ≤ f (x) ≤ M
In the previous example:
0 ≤ x ≤ 1 ⇒ 5 ≤ f (x) ≤ 7
f is bounded above by 7 and below by 5.

3.1.4 The parity of a function

A function f (x) is said to be **even** if, for every value x in its domain, the value of the
function is equal to the value of the function evaluated at −x. Mathematically,

f (x) is even ⇔ ∀x ∈ D, f (x) = f (−x)

Graphically, the graph of an even function is symmetric with respect to the y-axis (the vertical
axis). This means that if you take a point (x, y) on the graph of the function, the corresponding
point (−x, y) is also on the graph. Even functions exhibit this symmetry about the y-axis.

Some classical examples of even functions include:

- f (x) = x2 (the square function)


- f (x) = |x| (the absolute value function)
- f (x) = cos(x) (cosine of x)

On the other hand, a function f (x) is said to be **odd** if, for every value x in its domain, the
value of the function is the negative of the value of the function evaluated at −x. Mathemati-
cally,
f (x) is odd ⇔ ∀x ∈ D, f (−x) = − f (x)

Graphically, the graph of an odd function has axial symmetry about the origin (the point (0, 0)).
This means that if you take a point (x, y) on the graph of the function, the corresponding point
(−x, −y) is also on the graph. Odd functions exhibit this symmetry about the origin.

Some classical examples of odd functions include:


- f (x) = x3 (the cubic function)
- f (x) = sin(x) (sine of x)

In summary, a function is even if it is symmetric about the y-axis, and a function is odd if
it is symmetric about the origin.

31
(ESNN)S.Azoug CHAPTER 3. REAL FUNCTIONS OF A REAL VARIABLE
3.1.5 Periodic Functions

A function f : E −→ F is said to be periodic if there exists a real number P such that:

∀x ∈ E, x + P ∈ E, f (x + P) = f (x)
Examples:
1) x → sin(x) has a period of 2π.
2) x → cos(x) has a period of 2π.
3) x → tan(x) has a period of π.
4) x → x − bxc has a period of 1.

3.2 Limits
A function f defined in the neighborhood of x0 has a limit l as x approaches x0 if, as x gets
arbitrarily close to x0 , f (x) approaches l. This definition can be written as follows:

Definition 3.2.1.

lim f (x) = l ⇔ ∀ε > 0, ∃α > 0, ∀x ∈ V : 0 < |x − x0 | < α ⇒ | f (x) − l| < ε


x→x0

**4.** lim x→x0 f (x) = −∞ means that for any value A < 0, there exists a α > 0 such that for all
x ∈ I where |x − x0 | < α, f (x) < A.

**5.** lim x→+∞ f (x) = l means that for any ε > 0, there exists a α > 0 such that for all x ∈ I
where x > α, | f (x) − l| < ε.

**6.** lim x→−∞ f (x) = l means that for any ε > 0, there exists a α < 0 such that for all x ∈ I
where x < α, | f (x) − l| < ε.

**7.** lim x→a f (x) = l means that for any ε > 0, there exists a α > 0 such that for all x ∈ I
where 0 < |x − a| < α, | f (x) − l| < ε.

**Note:** This definition is mainly used for proving limit properties. In practice, other tech-
niques are often used.
**Left and Right Limits**

The definitions and propositions relate to the concept of the limit of a function f (x) as x ap-
proaches x0 , either from below (left) or from above (right).
**Definition :**
1. **Left Limit (lg ):** We say that the function f has a left limit lg as x approaches x0 from the
left (or from smaller values) if, for any ε > 0, there exists a α > 0 such that for all x ∈ I (where

32
(ESNN)S.Azoug 3.2. LIMITS
I is an interval), if x0 − α < x < x0 , then | f (x) − lg | < ε. This is denoted as lim x→x0− f (x) = lg .

2. **Right Limit (ld ):** We say that the function f has a right limit ld as x approaches x0 from
the right (or from larger values) if, for any ε > 0, there exists a α > 0 such that for all x ∈ I, if
x0 < x < x0 + α, then | f (x) − ld | < ε. This is denoted as lim x→x0+ f (x) = ld .

**Propositions:**
1. If f has a limit l as x approaches x0 , both from the left and from the right, then lim x→x0− f (x) =
lim x→x0+ f (x) = l.

2. If f has a left limit lg and a right limit ld such that lg = ld , then lim x→x0 f (x) = lg = ld .

3. If both left limit lg and right limit ld exist but are different, then f does not have a limit as x
approaches x0 .

These definitions and propositions are fundamental in mathematical analysis as they precisely
define what it means for a function to have a limit at a given point. This is crucial for under-
standing the behavior of functions at specific points and for performing calculations related to
continuity and differentiability of functions.

**Limits of Compositions**
The **theorem of the limit of a composite function** states that :

Theorem 3.2.1. colorblue


if f (x) approaches L as x approaches a, and if g(x) approaches M as x approaches L, then the
composition g( f (x)) approaches M as x approaches a.

Formally, if lim x→a f (x) = L and lim x→L g(x) = M, then lim x→a g( f (x)) = M.

This theorem is extremely useful in analysis because it allows us to calculate limits of complex
functions by breaking down these functions into simpler compositions and utilizing the limits
of individual parts. This greatly simplifies the calculation of limits in situations where functions
become complicated or involve multiple terms.
Let’s consider an example to illustrate this theorem:

Let’s consider the functions f (x) = x2 and g(x) = sin(x). We want to find the limit of g( f (x))
as x approaches 2.

1. **Limit of f (x) as x approaches 2**: lim x→2 f (x) = lim x→2 x2 = 22 = 4.

33
(ESNN)S.Azoug CHAPTER 3. REAL FUNCTIONS OF A REAL VARIABLE
2. **Limit of g(x) as x approaches 4** (since f (2) = 4):
lim x→4 g(x) = lim x→4 sin(x) = sin(4).

3. **Composition of Limits**:
lim x→2 g( f (x)) = g( f (2)) = g(4) = sin(4).

In this example, we used the theorem of the limit of a composite function to find the limit of
g( f (x)) by using the limits of f (x) and g(x) individually.

This theorem is a powerful tool in mathematical analysis because it enables us to understand


the behavior of complex functions in terms of the behavior of the individual parts of these
functions.

3.2.1 Operations on Limits


Let f and g be two functions defined on an interval I in R. Let x0 be an element of I. Suppose
lim x→x0 f (x) = l and lim x→x0 g(x) = l0 exist, then:
1) lim x→x0 ( f (x) + g(x)) = l + l0
2) lim x→x0 ( f (x) · g(x)) = l · l0
3) lim x→x0 (α · f (x)) = α · l (where α is a constant)
4) lim x→x0 | f (x)| = |l|

Proof: 1 Show that : lim x→x0 f (x) + g(x) = l + l0 lim x→x0 f (x) = l ⇔ ∀ε > 0, ∃α1 > 0, ∀x ∈ V :
0 < |x − x0| < ε ⇒ | f (x) − l| < ε/2.
lim x→x0 g(x) = l0 ⇔ ∀ε > 0, ∃α2 > 0, ∀x ∈ V : 0 < |x − x0| < ε ⇒ | f (x) − l0 | < ε/2.
We take α = min(α1, α2),Using triangular inequality lim x→x0 f (x) + g(x) = l0 + l ⇔ ∀ε >
0, ∃α > 0, ∀x ∈ V : 0 < |x − x0| < ε ⇒ | f (x) + g(x) − (l + l0 )| 6 | f (x) − l| + |g(x) − l0 | < ε.

Rest ith the same manner.

3.2.2 Theorems on Limits


Theorem 3.2.2. If a limit of f exists, then it is unique.

Theorem 3.2.3. Let f be a function defined in the neighborhood of x0 . Then, lim x→x0 f (x) = l
if and only if for any sequence (Un ) converging to x0 , limn→∞ f (Un ) = l.
**Remark:** This theorem is often used to demonstrate the non-existence of a limit. It is
sufficient to find two sequences converging to x0 such that limn→∞ f (Un ) , limn→∞ f (Vn ).
**Example:** To show that the limit of lim x→∞ sin(x) does not exist, it suffices to take Un = nπ
and Vn = (2n + 1)2π.
Theorem 3.2.4. Let f and g be two functions defined in a neighborhood of x0 such that
lim x→x0 f (x) and lim x→x0 g(x) exist. If f (x) 6 g(x), then lim x→x0 f (x) 6 lim x→x0 g(x).

34
(ESNN)S.Azoug 3.3. CONTINUITY
3.2.3 Calculating Limits
Indeterminate Forms:
These are forms where it is not possible to conclude directly. This leads to resolving the inde-
terminacy using specific results and techniques for each case in general.

, ,
0 ∞
0 0
0∞ , 00 , 1∞ , ∞ − ∞, ∞0

**Note:** Remarkable Limits :


1)lim x→0 sin(x)
x
=1
2)lim x→0 x = 1
tan(x)

3)lim x→x0 1−cos(x)


x2
= 21
x
4)lim x→x0 e x−1 = 1
5)lim x→x0 ln(x+1)
x
=1
6)lim x→x0 x = 1
sinh(x)

7)lim x→x0 cosh(x)−1


x2
= 21
8)lim x→0 tanh(x)
x
=1

3.3 Continuity

3.3.1 Definitions
Definition 3.3.1. f is continuous at x0 if: lim x→x0 f (x) = f (x0 )
or using the definition :

lim x→x0 f (x) = f (x0 ) ⇔ ∀ε > 0, ∃α > 0, ∀x ∈ V : 0 < |x − x0 | < ε ⇒ | f (x) − f (x0 )| < ε.

**Note:** 1) If a function f is not defined at x0 , then it is not continuous at x0 .


2) The left and right limits determine the left and right continuity at x0 . Therefore,

lim x→<x0 f (x) = lim x→>x0 f (x) = f (x0 )

3.3.2 Continuity Extension


Let f be a function defined and continuous on I − x0 . If lim x→<x0 f (x) exists, we say f is
extendable by continuity at x0 . The function defined by f˜ = f (x) for x , x0 and f˜(x0 ) =
lim x→x0 f (x) is the extension of f .

 f (x)
 if (x , x0 )
f˜(x) = 

 f (x0 )
 if (x = x0 )

35
(ESNN)S.Azoug CHAPTER 3. REAL FUNCTIONS OF A REAL VARIABLE

Figure 3.1: •

**Example:** f (x) = sin(x)


x
can be extended at 0 because f (0) = lim x→0 sin(x)
x
= 1.

sin(x)
 if (x , 0)
f˜(x) = 

 x
1
 if (x = 0)

3.3.3 Global Continuity


A function f defined on I in R is said to be continuous on I if it is continuous at every point in
I.
Example:
f is continuous on [a, b] if it is continuous at every point in (a, b) and is continuous from the
right at a and from the left at b.

3.3.4 Operations on Continuous Functions


If f and g are two functions defined and continuous in the neighborhood of x0 , then: ( f + g),
( f · g), (α · f + β · g), | f |, gf with g(x0 ) , 0 are continuous at x0 .
**Note:** The continuity of standard functions (sin(), ln(), ...) is assumed.

3.3.5 Theorems on Continuous Functions


Theorem 3.3.1. .(Sequential Criterion for Continuity)*
Let f be a function defined on an interval I in R, and x0 be an element of I. f is continuous at

36
(ESNN)S.Azoug 3.3. CONTINUITY
x0 if and only if for any sequence (Un ) converging to x0 , the sequence f (Un ) converges to f (x0 ).

**Remark:** This theorem is primarily used to demonstrate that a function is not continuous
at a point x0 . To do this, it suffices to find a convergent sequence approaching x0 such that
f (Un ) does not converge to f (x0 ).

Theorem 3.3.2. .(Boundedness of Continuous Functions)*


Any function continuous on a closed and bounded interval [a; b] is bounded.

Example: Consider the function f (x) = ln(x) on the interval (0, 1]. It is continuous and
bounded above by 1, but it is not bounded below at 0.

Theorem 3.3.3. .(Heine’s Theorem)*


For any function continuous on a closed and bounded interval I = [a; b], the function attains
at least once its upper and lower bounds on this interval. In other words, there exist α and β in
I such that:

f (α) = max x∈I f (x) = sup x∈I f (x), f (β) = min x∈I f (x) = inf x∈I f (x)

Theorem 3.3.4. .(Intermediate Value Theorem, IVT)*


Let f be a continuous function on a closed and bounded interval I = [a; b]. If f (a) · f (b) 6 0,
then the equation f (x) = 0 has at least one real root in I.

Theorem 3.3.5. . (Intermediate Value Theorem, IVT)*


Let I be an interval in R and f be a continuous function on I. Let a and b be two elements in I
such that a < b.

∀λ, f (a) 6 λ 6 f (b), ∃r ∈]a; b[, such that f (r) = λ.

Image of an Interval

Theorem 3.3.6. .(Image of an Interval)*


Let f be a continuous function on a closed and bounded interval I = [a; b]. Then the image
f (I) = f ([a; b]) is a closed and bounded interval.

**Note:**
The variation table is often used to determine the image of an interval as follows:

f ([a; b]) = [min x∈I f (x); max x∈I f (x)].

37
(ESNN)S.Azoug CHAPTER 3. REAL FUNCTIONS OF A REAL VARIABLE
3.4 The derivative of a function
The concept of derivative of a function is a fundamental idea in mathematics, especially in
calculus. Here is a definition of the derivative of a function:
**Derivative of a function at a point:** The derivative of a function f (x) at a point a is the
instantaneous rate of change of the function at point a. It is denoted as f 0 (a) or ddxf (a) and is
defined as follows:

f (a + h) − f (a)
f 0 (a) = limh−→0
h

In other words, the derivative of f at a is the limit of the change in f (x) as x approaches a,
where h is a small change in x. This measures how much the function f (x) changes instantly at
a.
**Derivative of a function:** The derivative of a function f (x), denoted as f 0 (x) or ddxf , is a new
function that gives the instantaneous rate of change of f for each value of x in the domain of
f . In other words, the derivative provides the rate at which f is changing at each point in its
domain.
The derivative characterizes the slope of the tangent to the graph of the function at each point.
A function is said to be differentiable over an interval if it has a derivative for every point in
that interval.
The derivative has numerous applications in mathematics, science, engineering, and many other
fields. It is used to understand change and motion, solve optimization problems, and in many
other practical applications.

3.4.1 The derivative and the continuity of a function


If a function f is differentiable at a point a, it means that the following limit exists:

f (a + h) − f (a)
f 0 (a) = lim
h→0 h

If this limit exists, it implies that the function f is continuous at a. In other words, if a function
is differentiable at a point, then it is continuous at that point.

Furthermore, if f is differentiable at a point a, then the function E(h) = f (a+h)−


h
f (a)
is also con-
tinuous at a, because E(h) is essentially the function representing the rate of change of f , and
it is derived at a.

Example : x −→ |x| this function is continouos but not differentiable at 0.

38
(ESNN)S.Azoug 3.4. THE DERIVATIVE OF A FUNCTION
3.4.2 Operations on the derivative function
Operations on the derivative function follow certain important rules in differential calculus.
Here are some of the basic rules for operations on derivative functions:

1. **Sum of functions:** If f (x) and g(x) are differentiable functions, then ( f (x) + g(x))0 (the
derivative of the sum) equals f 0 (x) + g0 (x).

2. **Difference of functions:** If f (x) and g(x) are differentiable functions, then ( f (x)−g(x))0
(the derivative of the difference) equals f 0 (x) − g0 (x).

3. **Product of functions:** If f (x) and g(x) are differentiable functions, then ( f (x) · g(x))0
(the derivative of the product) is given by the product rule: ( f (x)·g(x))0 = f 0 (x)·g(x)+ f (x)·g0 (x).

4. **Quotient
0 of functions:** If f (x) and g(x) are differentiable functions,
 f (x) 0 fand g(x) , 0, then
 f (x) 0 (x)·g(x)− f (x)·g0 (x)

g(x)
(the derivative of the quotient) is given by the quotient rule: g(x) = (g(x))2
.

5. **Composition of functions:** If f (x) is differentiable at x and g(x) is differentiable at


f (x), then the derivative of the composition (g( f (x)))0 is given by the chain rule: (g( f (x)))0 =
g0 ( f (x)) · f 0 (x).

These rules are essential for calculating derivatives of complex functions using basic operations
on derivative functions.
Often, we need to calculate the nth derivative using Leibniz’s Formula: If f and g are n times
differentiable, then we have:

( f · g)(n) =
Pn k (k) (n−k)
k=0 C n f g

Exercice 02
Donnez les dérivées nieme des fonctions élémentaires suivantes :

xn , sin(x), cos(x), tanh(x), e x , e−x , ln(x), sinh(x), cosh(x)


Correction

f(x) f (n) (x)


α!
xα (α−n)!
.xα−n
sin(x) cos(x) , n=4.k+1,− sin(x) ,n=4.k+2, − cos(x) , n=4.k+3,sin(x) ,n=4.k
cos(x) − sin(x) , n=4.k+1,− cos(x) ,n=4.k+2, sin(x) , n=4.k+3,cos(x) ,n=4.k
ex ex
e−x (−1)n e−x
e x +(−1)n e−x
cosh(x) 2
e x −(−1)n e−x
sinh(x) 2
(−1)n−1 .(n−1)!
ln(x) xn

39
(ESNN)S.Azoug CHAPTER 3. REAL FUNCTIONS OF A REAL VARIABLE
Exercice 03
Calculer la dérivée de la fonction suivante : f (x) = sin(etan(x ) )
2

Correction

(sin(etan(x ))0 = cos(etan(x ) ). cos2x


2 2 2
tan(x )
2 (x2 ) e

Figure 3.2: Mean Value Theorem

3.4.3 Rolle’s Theorem


Theorem 3.4.1. (Rolle’s Theorem)
If f is continuous on [a, b] (with a < b), differentiable on ]a, b[, and f (a) = f (b), then there
exists at least one point c in ]a, b[ such that f 0 (c) = 0.

Application
Let’s consider the function f (x) = x2 − 4x + 3 on the closed interval [1, 3].
1. **Continuity:** The function f (x) is a polynomial, so it is continuous everywhere.
2. **Differentiability:** The function f (x) is a polynomial, so it is differentiable everywhere.
3. **Endpoints:** f (1) = 12 − 4(1) + 3 = 0 and f (3) = 32 − 4(3) + 3 = 0.
Since f (x) is continuous on [1, 3], differentiable on (1, 3), and f (1) = f (3) = 0, Rolle’s The-
orem guarantees that there exists at least one number c in the open interval (1, 3) such that
f 0 (c) = 0.
To find c, we need to find the derivative of f (x) and solve for x:
f 0 (x) = 2x − 4
Setting f 0 (x) to zero and solving for x:
2x − 4 = 0

40
(ESNN)S.Azoug 3.4. THE DERIVATIVE OF A FUNCTION

Figure 3.3: Rolle’s Theorem

2x = 4
x=2
So, according to Rolle’s Theorem, there exists at least one c in the open interval (1, 3) (in this
case, c = 2) such that f 0 (c) = 0.

3.4.4 Mean Value Theorem or Finite Increments Theorem


Theorem 3.4.2. (Mean Value Theorem or Finite Increments Theorem)
If f is continuous on the closed interval [a, b] (with a < b) and differentiable on the open
interval ]a, b[, then there exists at least one point c in ]a, b[ such that:

f (b) = f (a) + (b − a) f 0 (c)

Remark* It is easy to provide a geometric interpretation of this theorem: we have the equiva-
lence:
f (b) − f (a)
f (b) = f (a) + (b − a) f 0 (c) ⇐⇒ f 0 (c) =
b−a
This ratio represents the slope of the line connecting the points (a, f(a)) and (b, f(b)), and f 0 (c)
is the slope of the tangent line to the graph of f at the point (c, f(c)).
Certainly! The Mean Value Theorem (MVT) states that if a function f (x) is continuous on the
closed interval [a, b] and differentiable on the open interval (a, b), then there exists at least one
number c in the open interval (a, b) such that the instantaneous rate of change (the derivative
f 0 (c)) is equal to the average rate of change of the function over the interval [a, b]. Mathemati-
cally, this can be expressed as:

f (b) − f (a)
f 0 (c) =
b−a

41
(ESNN)S.Azoug CHAPTER 3. REAL FUNCTIONS OF A REAL VARIABLE
1 Rb
F(c) = F(t).dt
b−a a

Application Here’s an example to illustrate the Mean Value Theorem:


Consider the function f (x) = x2 on the interval [1, 3].
1. **Continuity and Differentiability:** f (x) = x2 is a polynomial function, so it is continuous
and differentiable everywhere.
2. **Endpoints:** f (1) = 12 = 1 and f (3) = 32 = 9.
According to the Mean Value Theorem, there exists at least one c in the open interval (1, 3)
such that:

f (3) − f (1) 9 − 1
f 0 (c) = = =4
3−1 2
To find c, we can find the derivative of f (x):

f 0 (x) = 2x

Setting f 0 (x) to 4 and solving for x:

2x = 4
x=2

So, according to the Mean Value Theorem, there exists at least one c in the open interval (1, 3)
(in this case, c = 2) such that the instantaneous rate of change of the function f 0 (c) = 4 is equal
to the average rate of change of the function over the interval [1, 3].

This theorem has practical applications, especially in physics and engineering, where it is used
to find average quantities such as average velocity, average temperature, or average power over
a given interval.

3.4.5 Generalized mean value theorem


The generalized mean value theorem, also known as L’Hôpital’s rule, states the following:

Let f (x) and g(x) be two functions defined on an open interval (a, b), except possibly at a point
c within that interval. If the following conditions are met:

1. f (x) and g(x) are continuous on (a, b),


2. f (x) and g(x) are differentiable on (a, b),
3. g0 (x) , 0 for all x in (a, b),
4. lim x→c f (x) = lim x→c g(x) = 0 or ±∞, or lim x→c f (x) = lim x→c g(x) = ∞ (that is, the
functions f (x) and g(x) have indeterminate forms 00 or ∞

at the point c),

42
(ESNN)S.Azoug 3.4. THE DERIVATIVE OF A FUNCTION
Then, there exists at least one real number c in the interval (a, b) such that:

f (x) f 0 (x)
lim = lim 0
x→c g(x) x→c g (x)

This theorem is extremely useful for evaluating limits of functions in cases where the limits
indicate an indeterminate form of 00 or ∞

. L’Hôpital’s rule provides a method to calculate these
limits by taking the derivatives of the functions f (x) and g(x) and forming the ratio of these
derivatives.

3.4.6 Optimality and Applications

**Definition: Global Extremum**

We say a function f (x) has:

- **Global Maximum at x = c**, if f (x) ≤ f (c) for every x in the domain of f ;


- **Global Minimum at x = d**, if f (d) ≤ f (x) for every x in the domain of f .

We call f (c) the **global maximum of f (x)** and f (d) the **global minimum of f (x)**.

In other words, a global maximum is the **highest point** on the graph of the function within
its domain, and a global minimum is the **lowest point** on the graph of the function within
its domain.

If the search for a minimum is not limited to a closed bounded interval, the following re-
sult is also applicable:

**Definition:** A function f is called **coercive over R** if ”it tends to infinity as x tends to
infinity”:
lim f (x) = +∞
|x|→+∞

Or coercive over an open interval ]a, b[ if:

lim f (x) = +∞
x→a+

lim f (x) = +∞
x→b−

43
(ESNN)S.Azoug CHAPTER 3. REAL FUNCTIONS OF A REAL VARIABLE
**Theorem:** Let Ω be an open interval.
Every function that is continuous and coercive on Ω achieves its minimum on Ω.

**Theorem:** If f is a function defined and differentiable on an open interval I and if f has


an extremum at a point x∗ in I, then f 0 (x∗ ) = 0.

**Note:** The converse of this theorem is false (the function x 7→ x3 has a derivative equal to
zero at 0, but it is not an extremum).
If f 0 (x∗ ) = 0, we say that x∗ is a critical point of f .
Extrema within the open interval I are to be found among the critical points.
If one is looking for an extremum on a closed interval [a, b], the analysis will be performed on
the open interval ]a, b[ and then compared to f (a) and f (b).

Optimality of seconde order

**Theorem:** Let f be a function that is twice differentiable on an open interval I, and x∗ ∈ I


be a critical point of f . Then: - If f 00 (x∗ ) > 0, f has a strict local minimum at x∗ . - If f 00 (x∗ ) < 0,
f has a strict local maximum at x∗ . - If f 00 (x∗ ) = 0, no conclusion can be made.

44
(ESNN)S.Azoug 3.5. SERIES OF EXERCISES ON FUNCTIONS
3.5 Series of exercises on functions
Exercise : 01.(Domain of definition)
Determine r the domain of definition of each of the following functions
1−x
1. f1 (x) =
4 − x2
2. f2 (x) = ln(x2 − 3x + 2)
ln(x2 − 3x + 2)
3. f3 (x) =
|x2 − 3|
4.
1

si(x > 0)


f4 (x) = 

2−x

x − 3x + 2 si(x < 0)

 2

Exercise : 02.
Determine the domain of definition of each of the following functions :
1. f2 (x) = 3.cos3 (x)
2. f2 (x) = tan(x)

Exercise : 03.

”Calculate the following limits.”:


x+2 x3 −2x2 +3 e x+1 ln(3x+1)
A. lim x−→0+ x2 ln(x)
; lim x−→+∞ x. ln(x)
; lim x−→+∞ x+2
; lim x−→0+ 2x

x+4 3 x+1
); lim x−→0+ (1 + x)ln(x) ; lim x−→+∞ ( xx2 +5
x x −1 3
B. lim x−→0+ ln(x+1)
; lim x−→−∞ 2
x+1
. ln( 2 +2
) x2 +1
1−x
√ q √ q √
x. ln(x2 +1) n −1
C. lim x−→+∞ 1+e x−3
; lim x−→1 xx−1 ; lim x−→+∞ ( x+ x2 +1− x+ x2 − 1)

Exercise: 04.(Domain of Definition)


Determine the domain of definition for each of the following functions:
ln(x2 − 3x + 2)
r
1−x
1. f1 (x) = 2. f 2 (x) = ln(x 2
− 3x + 2) 3. f 3 (x) = 4.
4 − x2 |x2 − 3|

1
 if x > 0
f4 (x) = 

 2−x
 x2 − 3x + 2 if x < 0

Exercise: 05.
Determine the domain of definition and the limits at the boundaries for the following functions:

45
(ESNN)S.Azoug CHAPTER 3. REAL FUNCTIONS OF A REAL VARIABLE
1. f (x) = ln(ln(x))
x
2. f (x) = eex −1
+1
1
3. f (x) = exx
4. f (x) = ln(|x|)

Exercise: 06.(Limits at x = 0)
Calculate the following limits:

sin(2x)
1. lim x→0 x
sin(2x)
2. lim x→0 sin(3x)
tan(ax)
3. lim x→0 sin(bx)
x−sin(ax)
4. lim x→0 x+sin(3x)
x2
5. lim x→0 sin(πx)
 
6. lim x→0 x sin 1x
Exercise: 07. Calculate f oh and ho f for the following functions and draw conclusions.

1. f (x) = x2 , h(x) = ln(x)


2. f (x) = x2 , h(x) = e x
3. f (x) = ln(x),
√ h(x) = e x
4. f (x) = x, h(x) = x2

Exercise: 08. (Parity of Functions)


Determine the parity of the following functions:

x
1. f (x) = eex −1
+1
2. f (x) = 1 + x2 + cos(3x)
3. f (x) = x + x3 + tan(2x)
f (x) = x +sin
2 2
(x)
4. |x|+6
x2
5. f (x) = sin(πx) 
6. f (x) = x sin 1
x
Exercise: 09.
Determine the values of a and b for f to be continuous on R.


 x2 − 3x,
 if x < 2
f (x) = 

2x + bx,
 if x ≥ 2

46
(ESNN)S.Azoug 3.5. SERIES OF EXERCISES ON FUNCTIONS




eax , if x < 1
g(x) =  if x = 1

2,



 x2 + b2 , if x > 1

Exercise: 10.(Continuity)
1. Study the continuity of the function f at the point x0 = 1:

f (x) = |x − 1|

2. Study the continuity of the function f at the point x0 = 0:


  
 x 2
cos 1
, if x , 0
f (x) = 

 x
0,
 if x = 0

3. For what value of a is f continuous at x = 0?


√ √
 1+x− 1−x
, if x , 0
f (x) = 

 x
2
3a − 27,
 if x = 0

4. For which values of α and β is f continuous at x = 2?



 x2 + αx − β, if x ≤ 2

f (x) = 

2x + β,
 if x ≥ 2

Exercise: 11.(Continuity Extension)


Can the following functions be extended to 0?

1. f5 (x) = x ln(|x|)
2. f5 (x) = x − |x|x  
3. f5 (x) = sin(x) sin 1x

Exercise: 12.
Let f be a function from [a, b] to R that is continuous, such that f (a) = f (b). Prove that the
function g defined by:
!
b−a
g(t) = f t + − f (t)
2

47
(ESNN)S.Azoug CHAPTER 3. REAL FUNCTIONS OF A REAL VARIABLE
vanishes at least at one point c in [a, a+b
2
].

Exercise: 13.(Differentiability)
Calculate the derivatives of the following functions:
√3
**A.** 1. f (x) = x2 + x + 1
2. f (x) = x x√
3. f (x) = e x+sin(x)
4. f (x) = ln(3x+1)
2x

**B.** 1. f (x) = a x!− 1


x3 + 4
2. f (x) = ln
1 − x2
3. f (x) = (1 + x)ln(x)
x
4. f (x) = xx2 +5
3
+2

**C.** 1. f (x) = tanh(x2 + 1)


2. f (x) = sinh2 (x)
3. f (x) = arctan(3x)
4. f (x) = arcsin(2x + 5)

Exercise: 14.(Differentiability Study)**


Study the differentiability of the following functions on their domains of definition and provide
the expressions for their derivatives.

1. 
 1+|x| , if x < 2
 x
f (x) = 

2x,
 if x ≥ 2

2.
 1+e 1x ,
 x
 if x , 0
g(x) = 

0, if x = 0

Exercise: 15. Consider the function f defined by:


 2
 x , if x , 0
 sin(x )
f (x) = 

0,
 if x = 0

1. Show that f is differentiable on R.


2. Calculate the derivative of f .

48
(ESNN)S.Azoug 3.5. SERIES OF EXERCISES ON FUNCTIONS
3. Prove that f is of class C 1 on its domain of definition.
Exercise: 16.
Calculate the nth derivatives of the following functions:

1. f (x) = (x2 − x + 2)e x


2. f (x) = cos2 (x) sin(x)
1
3. f (x) = 2
x −1

Exercise: 17.

1 Pn
Show that if f (x) = √ , then f (n) (x) = where Pn is a polynomial of degree n.
(x2 + 1)n+ 2
1
x2 + 1
Find a relationship between Pn and Pn+1 .

Exercise: 18.(Intermediate Value Theorem)**

Let f be the function defined on [1, +∞[ by:

fn (x) = xn − x − 1, n > 2

A. Show that there exists xn ∈ [1, 1 + n1 ] such that fn (xn ) = 0.

B. Show that fn+1 (xn ) > 0.

C. Deduce that the sequence (xn ) is decreasing and converges to a limit l, and find l.

Exercise: 19.(Intermediate Value Theorem)**

A. Show that for all real x and y, the following inequality holds:

| sin(x) − sin(y)| 6 |x − y|.

B. Show that for all positive real x, the following inequalities hold:
x
< ln(x + 1) < x.
x+1

49
(ESNN)S.Azoug CHAPTER 3. REAL FUNCTIONS OF A REAL VARIABLE
Exercise: 20.(Intermediate Value Theorem)

Let f : [a, b] → R be a continuous function. Assume f is differentiable on a, b and for all


x ∈ [a, b], f (x) > 0.

Show that there exists c ∈]a, b[, such that:


f (a) f 0 (c)
= e(b−a) f (c)
f (b)

50
Chapter 4
Common functions and their inverse

4.1 Inverse Function


Definition 4.1.1. An application defined from I to J is called bijective if and only if: 1) f is
surjective, meaning:

∀y ∈ J, ∃x ∈ I such that y = f (x)

2) f is injective, meaning:

∀x, x0 ∈ I, f (x) = f (x0 ) ⇒ x = x0 .

**Remarks:** 1) An application defined from I to J is bijective if and only if the equation


y = f (x) has a unique solution x in I for any fixed y in J. 2) A bijective application has an
inverse function denoted f −1 , which is defined from J to I as follows: x = f −1 (y).
Theorem 4.1.1. . (Bijection Theorem) If f is a continuous and strictly monotone function from
I to J: 1) f is a bijection from I to f (I) = J.
2) Its inverse f −1 is defined from J to I and is continuous and strictly monotone on J.

**Remark:** There exists a relationship between surjection and monotonicity, as well as be-
tween injection and continuity.
e x +e−x e x −e−x
**Applications:** Consider cosh(x) = 2
and sinh(x) = 2
. Show that they have inverse
functions.

4.1.1 Relation between function and the inverse


∀y ∈ D f −1 , f ( f −1 (y)) = y.

∀x ∈ D f, f −1 ( f (x)) = x

51
(ESNN)S.Azoug CHAPTER 4. COMMON FUNCTIONS AND THEIR INVERSE
4.1.2 The derivative of the inverse function

The derivative of the inverse function ( f −1 (x)) of a bijective function f (x) is given by the fol-
lowing formula:

( f −1 (x))0 = 1
f 0 ( f −1 (x))

This formula allows calculating the derivative of the inverse function at a given point.

It is crucial to note that this formula only works if the function f (x) is bijective (meaning it has
a well-defined inverse) and if f 0 ( f −1 (x)) is not equal to zero. The condition f 0 ( f −1 (x)) , 0 is
important because it avoids division by zero in the formula.

4.2 Common Functions

4.2.1 Trigonometric Functions

4.2.2 x → cos(x)

• Domain of definition D f = R

• Periodic with period 2π

• Parity: even ∀x ∈ D f , cos(−x) = cos(x)

• Study interval E = [0; π]

• Derivative: continuous and differentiable on E, and we have (cos(x))0 = − sin(x)

• Bijection from E to [-1; 1].

• With f −1 (x) = cos−1 (x) = arccos(x) ∀x ∈ R, −1 ≤ cos(x) ≤ 1

π
x 0 π
2
− sin(x) − 0 −

1
cos(x)
0 -1

52
(ESNN)S.Azoug 4.2. COMMON FUNCTIONS
4.2.3 x → cos (x) = arccos(x)
−1

• Domain of definition D f = [−1; 1]


−1
• Derivative: continuous and differentiable on E, and we have (arccos(x))0 = √
1 − x2
• Bijection from [-1; 1] to [0; π]

• With f −1 (x) = cos(x) ∀x ∈ R, 0 ≤ arccos(x) ≤ π

Don’t forget that:

arccos(cos(x)) = x, x ∈ [0; π]

cos(arccos(x)) = x, x ∈ [−1; 1]

0.0 0.5 1.0 1.5 2.0 2.5 3.0

3 cos(x) 3
arccos(x)
x

2 2

1 1

0 0

-1 -1

0.0 0.5 1.0 1.5 2.0 2.5 3.0

Figure 4.1: •

53
(ESNN)S.Azoug CHAPTER 4. COMMON FUNCTIONS AND THEIR INVERSE
4.2.4 x → sin(x)
• Domain of definition D f = R

• Periodic with period 2π

• Parity: odd ∀x ∈ D f , sin(−x) = − sin(x)

• Study interval E = [− π2 ; π2 ]

• Derivative: continuous and differentiable on D f = R, and we have (sin(x))0 = cos(x)

• Bijection from E to [-1; 1].

• With f −1 (x) = sin−1 (x) = arcsin(x)

∀x ∈ R, −1 ≤ sin(x) ≤ 1

4.2.5 x → sin−1 (x)


• Domain of definition D f = [−1; 1]

• Derivative: continuous and differentiable on E, and we have

1
(arcsin(x))0 = √
1 − x2

• Bijection from [-1; 1] to [− π2 ; π2 ]

• With f −1 (x) = sin(x)

π
∀x ∈ [−1; 1], −π
2
≤ arcsin(x) ≤ 2

Don’t forget that:

arcsin(sin(x)) = x, x ∈ [− π2 ; π2 ]

sin(arcsin(x)) = x, x ∈ [−1; 1]

54
(ESNN)S.Azoug 4.2. COMMON FUNCTIONS

Figure 4.2: •

4.2.6 x → tan(x)
• tan(x) = sin(x)
cos(x)

• Domain of definition D f = R − { −π
2
+ 2kπ, +π
2
+ 2kπ}

• Periodic with period π

• Parity: odd ∀x ∈ D f , tan(−x) = − tan(x)

• Study interval E =] −π
2
; +π
2
[

• Derivative: continuous and differentiable on E, and we have

(tan(x))0 = 1 + tan2 (x)

• Bijection from E to ] − ∞; +∞[.

• With f −1 (x) = arctan(x)

4.2.7 x → arctan(x)
• Domain of definition D f = R

• Parity: odd

• Derivative: continuous and differentiable on D f = R, and we have

• (arctan(x))0 = 1
1+x2

55
(ESNN)S.Azoug CHAPTER 4. COMMON FUNCTIONS AND THEIR INVERSE
• Bijection from D f = R to ] −π
2
; +π
2
[.

• With f −1 (x) = tan(x)

Don’t forget that:

arctan(tan(x)) = x, x ∈ R − { −π
2
+ 2kπ; +π
2
+ 2kπ}

tan(arctan(x)) = x, x ∈ R

4.3 Hyperbolic Functions

4.3.1 x → cosh(x)
e x +e−x
• Domain of definition cosh(x) = 2

• Domain of definition D f = R

• Parity: even ∀x ∈ D f , cosh(−x) = cosh(x)

• Study interval E = [0; +∞[

• Derivative: continuous and differentiable on E, and we have

(cosh(x))0 = sinh(x)

• Bijection from E to E = [1; +∞[.



• With f −1 (x) = cosh−1 (x) = arg cosh(x) = ln(x + x2 − 1)


arg cosh(x) = ln(x + x2 − 1)

4.3.2 x → cosh−1 (x) = arg cosh(x)

• Domain of definition D f = [1; +∞[

• Derivative: continuous and differentiable on E, and we have

1
(arg cosh(x))0 = √
x2 − 1

56
(ESNN)S.Azoug 4.3. HYPERBOLIC FUNCTIONS
4.3.3 x → sinh(x)
e x −e−x
• Domain of definition sinh(x) = 2

• Domain of definition D f = R

• Parity: odd ∀x ∈ D f , sinh(−x) = − sinh(x)

• Derivative: continuous and differentiable on E, and we have

(sinh(x))0 = cosh(x)


• With f −1 (x) = sinh−1 (x) = arg sinh(x) = ln(x + x2 + 1)


arg sinh(x) = ln(x + x2 + 1)

4.3.4 x → sinh−1 (x) = arg sinh(x)

• Domain of definition D f = R

• Derivative: continuous and differentiable on E, and we have

1
(arg sinh(x))0 = √
x2 + 1

e x −e−x
4.3.5 x → tanh(x) = sinh(x)
cosh(x) = e x +e−x

• Domain of definition D f = R

• Parity: even ∀x ∈ D f , tanh(−x) = − tanh(x)

• Derivative: continuous and differentiable on E, and we have

(tanh(x))0 = 1 − tanh2 (x)

• Bijection from E to [−1; 1].

• With f −1 (x) = tanh−1 (x) = arg tanh(x)

∀x ∈ R, −1 < tanh(x) < 1

57
(ESNN)S.Azoug CHAPTER 4. COMMON FUNCTIONS AND THEIR INVERSE
4.3.6 Important relationships and identities
The hyperbolic functions have certain relationships and identities, similar to trigonometric
functions. Here are some of the key relationships:

Addition Formulas:

1. sinh(a + b) = sinh(a) cosh(b) + cosh(a) sinh(b)

2. cosh(a + b) = cosh(a) cosh(b) + sinh(a) sinh(b)

3. tanh(a + b) = tanh(a)+tanh(b)
1+tanh(a) tanh(b)

Double-Angle Formulas:
1. sinh(2x) = 2 sinh(x) cosh(x)

2. cosh(2x) = cosh2 (x) + sinh2 (x)

3. tanh(2x) = 2 tanh(x)
1+tanh2 (x)

Other Identities:
1. cosh2 (x) − sinh2 (x) = 1

2. tanh2 (x) = sech2 (x)

3. coth2 (x) = 1 + csch2 (x)

Reciprocal Relationships:
1. sec(x) = 1
cosh(x)

2. csch(x) = 1
sinh(x)

3. coth(x) = 1
tanh(x)

These formulas and relationships are useful for simplifying expressions involving hyperbolic
functions.

58
(ESNN)S.Azoug
4.4. SERIES OF EXERCISES ON COMMON FUNCTIONS AND INVERSE
4.4 Series of exercises on common functions and inverse
Exercise : 01.
Resolve the following equations : 1.ln(cosh(x)) = a , a ∈ R, a > 1
2.arccos(x) = 2arccos( 34 )
3. arcsin(x) = arcsin( 25 ) + arcsin( 35 )
4. arctan(2x) + arctan(x) = π

Exercise : 02.
Determine the domain of definition of each of the following functions :
1. f2 (x) = 5. sinh(x) − 3. cosh(x)
2. f2 (x) = sinh(2x)
sinh(x)

3. f2 (x) = 2. tanh(2x)
4. f2 (x) = 3.cosh3 (x)

Exercise : 03.
2 cosh 2(x) − sinh(2x)
Simplify the following expression calculate the limits on + − ∞
x − ln(cosh x) − ln(2)
Exercise: 04. (Parity of Functions)
Determine the parity of the following functions:

1. f (x) = 2. tanh(2x) − 3.sinh(3x)


2. f (x) = 3.cosh3 (x) + x2
3. f (x) = x + x3 + tanh(2x)

Exercise: 05.(Inverse Function)


Let f be defined as:





 x, if x < 2
f (x) =  x,

 2
if 1 6 x 6 4

 √
8 x, if x > 4

1. Plot the graph of f .


2. Is f continuous on R?
3. Show that f has an inverse function f −1 and provide its expression.
Exercise: 06.(Inverse)

Let f be the function defined by:


x
f (x) =
1 + |x|

59
(ESNN)S.Azoug CHAPTER 4. COMMON FUNCTIONS AND THEIR INVERSE
1. Show that f establishes a bijection from R to f (R).
2. Determine f (R) and f −1 .

60
Chapter 5
limit development

5.1 Comparison of Functions in the Vicinity of a Point. Lan-


dau Notations
Throughout this chapter, we will often be required to compare two functions in the vicinity of
a point with abscissa x0 . For this purpose, we introduce definitions using Landau notations.

5.1.1 Definitions and Properties

Definition 1: Let f and g be two functions defined in a neighborhood of the point x0 (except
possibly at x0 ). We say that f is negligible compared to g as x approaches x0 , denoted as
f = o(g), if
∀ε > 0, ∃δ > 0, ∀x 0 < |x − x0 | < δ =⇒ | f (x)| 6 ε · |g(x)| .
 

Definition 2: We say that f is dominated by g as x approaches x0 , denoted as f = O(g), if

∃k > 0, ∃δ > 0, ∀x 0 < |x − x0 | < δ =⇒ | f (x)| 6 k · |g(x)| .


 

The symbols o and O are called **Landau notations**.


Remarks: It follows from these definitions that if g does not approach zero in a neighborhood
of x0 : 1. f = o(g) ⇐⇒ lim x−→x0 g(x)
f (x)
= 0. 2. f = O(g) ⇐⇒ lim x−→x0 | g(x)
f (x)
| 6 k.

Particular case

: if g=1, then:

1. f = o(1) ⇐⇒ lim x−→xo f (x) = 0.

2. f = O(1) ⇐⇒ lim x−→xo | f (x) |6 k.

61
(ESNN)S.Azoug CHAPTER 5. LIMIT DEVELOPMENT
Therefore, we denote a function that approaches 0 as o(1),
and one that is bounded as O(1).

Examples of Applications:
x2
x2 = o(x) ⇐⇒ lim =0
x−→0 x

x2 · sin(x)
x2 · sin(x) = O(x) ⇐⇒ lim =0
x−→0 x
x · sin(x) = O(x)
in the vicinity of infinity.
**Exercise**
Show that: 1
1−x
= 1 + x +o(x)
 in the vicinity of zero.
Show that: 1
1−x
= x + O x2 in the vicinity of zero.
−1 1

Equivalent functions
**Definition**: Let f and g be two functions defined in a neighborhood of x0 (except possibly
at x0 ). We say that f is equivalent to g as x approaches x0 and denote it as f ∼ g, if f − g = o(g)
as x approaches x0 .

Equivalent

f ∼ g ⇐⇒ f − g = o(g) ⇐⇒ f − g = o( f )

f ∼ g ⇐⇒ Lim x−→x0 ( gf ) = 1

Proof f − g = f.o(1) ⇐⇒ g = f (1 + o(1)) ⇐⇒ f = g. 1+o(1)


1

Theorem 5.1.1. Let f, f1 , g, g1 be functions defined in a neighborhood of x0 . Suppose that:


f ∼ f1 , g ∼ g1 as x → x0 , and if lim x−→x0 gf exists, then lim x−→x0 gf11 also exists, and the two
limits are equal.

Proof
f1 (1 + o(1))
Write f
= and take the limit, then conclude.
g
g1 (1 + o(1))

Example
 sin2 (x)  1
Show that lim x−→0 tan2 (2x)
= .
4
62
(ESNN)S.Azoug 5.2. TAYLOR POLYNOMIALS
5.1.2 Properties of Equivalence
1. If f ∼ f1 and g ∼ g1 , then f g ∼ f1 g1

1 1
2. If f ∼ f1 and f , 0, f1 , 0, then f
∼ f1

f f1
3. If f ∼ f1 and g ∼ g1 , then g
∼ g1

4. If f ∼ f1 , then f α ∼ f1α (for positive f and f1 in the vicinity of x0 )

Proof ( exos)
**Note:** Be cautious, the summation is incorrect.

5.2 Taylor Polynomials


It is always desirable to replace complicated functions with simpler ones, like polynomials,
for ease of calculations in limits, integrals, etc. The Taylor formula provides us with these
polynomials.

Definition 5.2.1. The Taylor polynomial of order n generated by f at the point x0 is the poly-
nomial Pn of degree less than or equal to n, defined by:

(x − x0 )2 (2) (x − x0 )n (n)
Pn (x) = f (x0 ) + (x − x0 ) f 0 (x0 ) + f (x0 ) + . . . + f (x0 )
2! n!

Example 1

• Taylor polynomial of degree n for f (x) = e x at 0.

x2 xn
Pn (x) = 1 + x + + ... +
2! n!

Example 2

• Taylor polynomial of degree 3 for f (x) = sin(x) at 0.

x3
P3 (x) = x −
3!

Taylor Expansion of Order n in the Vicinity of Zero Definition Let f be a function defined in
the neighborhood of 0 (except possibly at 0). We say that f has a Taylor expansion at 0 of order
n if there exist constants a0 , a1 , . . . , an such that:

f (x) = a0 + a1 x + . . . + an xn + o(xn )

63
(ESNN)S.Azoug CHAPTER 5. LIMIT DEVELOPMENT

Figure 5.1: Taylor Polynomials (1st, 2nd, 3rd, 4th) for f (x) = e x

where f (x) = pn (x) + o(xn ) and pn (x) is the regular part with deg pn 6 n.

Taylor Formulas To write the Taylor expansion for C n+1 class functions, the following formula
is used:
n
f (x) = f (x0 ) + (x − x0 ) · f (1) (x0 ) + . . . + (x−x
n!
0)
· f (n) (x0 ) + Rn (x − x0 )
Rn (x − x0 ): remainder term of order n.

**First Form with Young’s Remainder**


(x − x0 )n (n)
f (x) = f (x0 ) + (x − x0 ) · f (1) (x0 ) + . . . + · f (x0 ) + o((x − x0 )n )
n!

**Second Form with Lagrange’s Remainder**

(x − x0 )n (n) (x − x0 )n+1 · f (n+1) (c)


f (x) = f (x0 ) + (x − x0 ) · f (1) (x0 ) + . . . + · f (x0 ) +
n! (n + 1)!
where c ∈]x0 , x[.
**Application** Write the dl3 (x0 = 1) of e x :
(x − 1)2 (2) (x − 1)3 (3)
f (x) = f (x0 ) + (x − 1) f (x0 ) +
(1)
f (x0 ) + f (x0 ) + o((x − 1)3 )
2! 3!
Taking n = 3 and x0 = 1, we get:
(x − 1)2 (x − 1)3
e x = e1 + (x − 1)e + e+ e + o((x − 1)3 )
2! 3!

64
(ESNN)S.Azoug 5.2. TAYLOR POLYNOMIALS
MacLaurin’s Formula When x0 = 0 in the Taylor-Lagrange formula, we obtain the MacLaurin
formula:

xn
f (x) = f (0) + x · f (1) (0) + . . . + n!
· f (n) (0) + o(xn )

Property Let f and g be two functions n times differentiable at a point x0 , and Pn ( f ) and Pn (g)
be their n-th order Taylor polynomials at x0 . Then:

• The n-th order Taylor polynomial for the sum f + g at point x0 is the polynomial Pn ( f ) +
Pn (g).

• The n-th order Taylor polynomial for the product f · g at point x0 is the sum of terms of
degree less than or equal to n of the polynomial Pn ( f ) · Pn (g).

• The n-th order Taylor polynomial for f (xk ) at point x0 is the polynomial Pn ( f (k) ).

• The (n − 1)-th order Taylor polynomial for f 0 at point x0 is the polynomial P0 ( f ).

Examples

x2
• The 2nd order Taylor polynomial for the sum e x + sin(x) at x0 = 0 is P = 1 + 2x + 2
.

• The 2nd order Taylor polynomial for the product e x · sin(x) at x0 = 0 is P = x + x2 .

• The 2nd order Taylor polynomial generated by e x at x0 = 0 is P = 1 + x2 .


2

• The 2nd order Taylor polynomial generated by cos(x) at x0 = 0 is P = (sin(x))0 =


3 2
(x − x6 )0 = 1 − x2 .

Figure 5.2: Trick for Taylor Series (S. Azoug)

5.2.1 Maclaurin Series of Elementary Functions


x2 x3 xn+1 n
• ex = 1 + x + 2!
+ 3!
+ ... + n!
x + O(xn )

• 1
1−x
= 1 + x + x2 + x3 + . . . + xn + O(xn )
x3 x5 (−1)2n+1 x2n+1 n
• sin(x) = x − 3!
+ 5!
+ ... + (2n+1)!
x + O(x2n+1 )

65
(ESNN)S.Azoug CHAPTER 5. LIMIT DEVELOPMENT
x2 x4 (−1)n x2n
• cos(x) = 1 − 2!
+ 4!
+ ... + (2n)!
xn + O(x2n )

x3 2x5
• tan(x) = x + 3
+ 15
+ ...

x3 x5 x2n+1 n
• sinh(x) = x + 3!
+ 5!
+ ... + (2n+1)!
x + O(x2n+1 )

x2 x4 x2n n
• cosh(x) = 1 + 2!
+ 4!
+ ... + (2n)!
x + O(x2n )

x3 2x5
• tanh(x) = x − 3
+ 15
+ ...

Figure 5.3: Trick for Dls Azoug

Figure 5.4: Trick for Dls Azoug

Figure 5.5: Trick for derivatives.

Application to common functions.

1.
xn (n)
f (x) = f (0) + x. f (1) (0) + ... + . f (0) + o(xn )
n!

66
(ESNN)S.Azoug 5.2. TAYLOR POLYNOMIALS
2.
xn 0
e x = e0 + x.e0 + ... + .e + o(xn )
n!

xn
3. e x = 1 + x + x2 + ... + n!
+ o(xn )

x3 (−1)n x2n+1
4. sin(x) = x − 3!
+ ... + (2n+1)!
+ o(x2n+2 )

x2 (−1)n x2n
5. cos(x) = 1 − 2!
+ ... + (2n)!
+ o(x2n+1 )

x2 (−1)n .xn
6. log(1 + x) = x − 2
+ ... + n
+ o(xn )

xn
7. 1
1−x
= 1 + x + x2 + ... + n!
+ o(xn )

x2
8. (1 + x)α = 1 + α.x + α(α − 1) + ... + (αn ).xn + o(xn )
2

x3
9. tan = x + + ...
3

x3 x2n+1
10. sh(x) = x + 3!
+ ... + (2n+1)!
+ o(x2n+2 )

x2 x2n
11. ch(x) = 1 + 2!
+ ... + (2n)!
+ o(x2n+1 )

Theorem (Uniqueness)
If f has a Taylor expansion of order n in the vicinity of 0, then this expansion is unique.
**Proof**
Suppose we can write:

f (x) = a0 + a1 x + . . . + an xn + o(xn )
and
f (x) = b0 + b1 x + . . . + bn xn + o(xn )
then take the difference and let x approach 0.

Corollary If f is an even function (resp: odd), and it has a Taylor expansion in the vicinity
of 0, its regular part is an even (resp: odd) polynomial.

67
(ESNN)S.Azoug CHAPTER 5. LIMIT DEVELOPMENT
**Important Note:**
A necessary condition for the existence of a Taylor expansion is that the limit at x0 = 0 must
exist.  
**Counterexample** f (x) = xn+1 sin xn+1 1
if x , 0 and f (0) = 1. Show that it has a Taylor
expansion of order n at 0, but f is not continuous at 0.

5.2.2 Algebraic Operations on Taylor Series


Theorem 5.2.1. Let f and g be two functions having Taylor expansions of order n in the
vicinity of 0. Then, the functions f + g, f · g have Taylor expansions of order n at 0, and the
same is true for f /g if lim x−→0 g(x) , 0.
Moreover, the regular part of the Taylor expansion of f + g is the sum of the regular parts of
the Taylor expansions of f and g;
the regular part of the Taylor expansion of f · g is obtained by keeping in the product only
the terms of degree less than or equal to n;
the regular part of the Taylor expansion of f /g is the quotient of order n in the division
following the increasing powers of the regular part of f by the regular part of g.

Exemples : Write the Taylor expansion of order 3 at zero for the following functions :
sin(x) + cos(x), sin(x). cos(x), cos(x)
sin(x)

Sum

x3
1. sin(x) = x − 3!
+ o(x3 )

x2
2. cos(x) = 1 − 2!
+ o(x3 )

x2 x3
3. sin(x) + cos(x) = 1 − 2!
+x− 3!
+ o(x3 )

x2 x3
4. sin(x) + cos(x) = 1 + x − 2!
− 3!
+ o(x3 )

Product

x3 x2
1. sin(x). cos(x) = (x − 3!
).(1 − 2!
) + o(x3 )

x3 x2
2. sin(x). cos(x) = (x − 6
).(1 − 2
) + o(x3 )

x3 x3 x5
3. sin(x). cos(x) = x − 2
− 6
+ 12
+ o(x3 )

68
(ESNN)S.Azoug 5.2. TAYLOR POLYNOMIALS
4x3 x3
4. sin(x). cos(x) = x − 6
− 6
+ +o(x3 )

Division

3
x− x6
1. sin(x)
cos(x)
= 2
1− x2

x3
2. sin(x)
cos(x)
= x+ 3
+ o(x3 )

Taylor Expansion of a Composite Function.

Theorem 5.2.2. Let f and u be functions having Taylor expansions of order n in the vicinity
of 0, with regular parts fn and un respectively. If u(0) = 0, the composite function f0 (u) has
a Taylor expansion of order n in the vicinity of 0. Its regular part is obtained by substituting
the regular part of the Taylor expansion of f with the regular part of u, retaining only the
powers of x up to order 6 n.

**Application:**
Write the Taylor expansions of the following functions: ecos(x) , esin(x) , log(1 + esin(x) ) up to order
5 at 0.
**Solution**

1. ecos(x) = e − 2e x2 − 6e x4 + o(x5 )

x2 x4 x5
2. esin(x) = 1 + x + 2
− 8
− 15
+ o(x5 )

x2 x3 3x4 x5
3. log(1 + esin(x) ) = log(2) + 2x + 8
− 12
− 64
− 240
+ o(x5 )

Integration of a Taylor Expansion.

Theorem 5.2.3. Let f : [−a, a] −→ R be an integrable function having a Taylor expansion


of order n in the vicinity of 0:

f (x) = a0 + a1 x + ... + an xn + o(xn ) = Pn (x) + xn ε(x)


Rx
Then, the function F : x 7→ 0 f (t)dt, x ∈ [−a, a] has a Taylor expansion of order n + 1 in the
vicinity of 0:
a1 x2 an xn+1
F(x) = a0 x + + ... + + xn+1 ∈ (x)
2 n+1

69
(ESNN)S.Azoug CHAPTER 5. LIMIT DEVELOPMENT
**Example** Find the dl3 (0) of the following function: arctan(x)
Correction
3 5
arctan(x) = x − x3 − x5 + o(x5 )

Derivation of a Taylor Expansion.

Theorem 5.2.4. If the function f is differentiable on [a, +a] and has a Taylor expansion of
order n in the vicinity of 0, and its derivative f 0 has a Taylor expansion of order n − 1 in
the vicinity of 0, then the regular part of the Taylor expansion of f 0 is the derivative of the
regular part of f .

Exercise:
Deduce the dl3 (0) of the following function: arctan2 (x).

Solution Write arctan0 (x) = 1


1+x2

5.2.3 Generalized Taylor Expansion


Let f be a function defined in the vicinity of 0. Suppose that f does not have a Taylor expan-
sion in the vicinity of 0, but the function xα · f (x) does have one. In this case, we can write:
xα . f (x) = a0 + a1 .x + ... + an .xn + o(xn ) d’où
a0 + a1 .x + ... + an .xn + o(xn )
f (x) =

Exercice :
sin(x)
Find the Talor expansion for : x
Solution

sin(x) x2 (−1)n x2n


=1− + ... + + o(x2n+1 )
x 3! (2n + 1)!
Applications to Limits and Asymptotes
Exercise 01.
Compute the limit as x approaches 0 for the following function:

(e2x − 1) sin(3x)
sinh2 (2x)
Solution

(e2x − 1) sin(3x) (2x)(3x) 3


lim 2
= lim =
x→0 sinh (2x) x→0 4x2 2

70
(ESNN)S.Azoug 5.2. TAYLOR POLYNOMIALS
Exercise 02.
Compute the limit as x approaches 0 for the following function:
(e2x −1)sin2 (3x) x
sh3 (x)
Solution
2 (3x)  x
2x.9x3 x
 2x  
lim x→0 (e −1)sin
3
sh (x)
= lim x→0 x 3 = 1.

5.2.4 Applications of optimization calculations for functions

The limited development is a powerful mathematical technique used to approximate complex


functions using simple polynomials. This technique can be applied in finding the optimal points
(minima or maxima) of a function. When searching for the optimum points for a function,
limited development can help approximate the function around these points to determine their
nature.
Here’s how limited development can be used to calculate the optima of a function:
**1. Approximating the Function Around the Optimal Point:** Suppose we are looking
for an optimum point for a function f (x) near x = a. We can use the limited development of
f (x) around x = a to obtain an approximation of the function in that neighborhood.

f 00 (a)
f (x) ≈ f (a) + f 0 (a)(x − a) + (x − a)2 + . . .
2!
**2. Determining the Optimum:** To find a minimum or maximum, we examine the behav-
ior of the approximate function near x = a. If the coefficient of (x − a)2 is positive, the function
is convex near x = a, indicating a local minimum. If the coefficient is negative, the function is
concave near x = a, indicating a local maximum.
**Example:** Consider the function f (x) = x3 − 6x2 + 9x + 2. We want to find the optimal
points of this function.
1. **Find the Derivatives:**
f 0 (x) = 3x2 − 12x + 9
f 00 (x) = 6x − 12

2. **Find Points Where f 0 (x) = 0:**

3x2 − 12x + 9 = 0

x2 − 4x + 3 = 0
(x − 3)(x − 1) = 0
x = 1 or x = 3

3. **Use Limited Development around x = 1 and x = 3 to Determine the Nature of the


Optima:** - For x = 1:
f 00 (1) = 6(1) − 12 = −6

71
(ESNN)S.Azoug CHAPTER 5. LIMIT DEVELOPMENT
The coefficient of (x − 1)2 is negative, indicating a local maximum at x = 1. - For x = 3:

f 00 (3) = 6(3) − 12 = 6

The coefficient of (x − 3)2 is positive, indicating a local minimum at x = 3.


In this example, the limited development around the optimal points (x = 1 and x = 3) helped
determine the nature of the optima (maximum at x = 1 and minimum at x = 3).

72
(ESNN)S.Azoug 5.3. SERIES OF EXERCISES ON TAYLOR EXPANSIONS
5.3 Series of exercises on Taylor expansions
Exercise: 01
What does f (x) = o(1) mean at x0 ?
What does f (x) = O(1) mean at x0 ?
x
Prove that 2x is not bounded as x → +∞.
x
Do we have 2x = O(1)?
(Hint: 2n > n(n + 1)/2, n > 2 by induction.)
Find conditions on a, b, and c to have:

• xa = o(xb ) as x → 0+

• xa = o(xb | ln(x)|c ) as x → 0+

• xa = o(e x ) as x → 0+

Exercise: 02
Show that if f is differentiable at x0 with f 0 (x0 ) , 0, then f (x) − f (x0 ) is equivalent to (x −
x0 ) f 0 (x0 ).
Deduce the equivalent functions for the following functions:
sin(x) at x = 0; tan(x) at x = 0; sh(x) at +∞; 1 − cos(x) at x = 0; ln(x+1) at x = 0;
Exercise: 03
Using Taylor series expansions, determine the functions equivalent to sin(x) at zero.
Exercise: 04
Determine the following Taylor series expansions:

• Dl3 (0) of sh(x), ch(x).

• Dl3 (0) of sh(x)ch(2x) - ch(x).

• Dl3 (0) of sh(x)ch(2x)−ch(x)


sh(x)
.

Exercise: 05(Class)
Determine the following Taylor series expansions:

• Dl3 ( π4 ) of sin(x)

• Dl4 (1) of cos(x)

• Dl3 (0) of ln(x+3)

• Dl3 (1) of e x

• Dl3 (0) of arctan(x + 3)

Exercise: 06

73
(ESNN)S.Azoug CHAPTER 5. LIMIT DEVELOPMENT
Show that:
esinh(x) − ecos(x)
!
e 2
ln sin(x) cosh(x)
= x + o(x2 )
e −e 1−e

Exercise: 07

Consider the odd function of class C ∞ defined by its Taylor series expansion up to order 6 as:
3 x5
f (x) = x− x6 + 120 +o(x6 ). Find a Taylor series expansion up to order 6 of its reciprocal function,
knowing that it is odd.

Exercise: 08
For each of the following functions, find an asymptote:
f (x) = (x + 1)e1/x , f (x) = x(ln(2x + 1) − ln(x)).

Exercise: 09

Compute the limits at 0 for the following functions:


x − arcsin(x)
f (x) = 3
, f (x) = ln(x+1)
tan(x)
, f (x) = sin12 (x) − x12 .
sin (x)
Exercise: 10(Taylor Series)**

Compute the following Taylor series expansions:

A. Taylor series expansion of f (x) = sinh(2x) at x = 0.

B. Taylor series expansion of f (x) = sinh(x) cosh(x) − cosh(x) at x = 0.

C. Taylor series expansion of f (x) = arctan(x) at x = 0.

E. Taylor series expansion of f (x) = argsinh(x) at x = 0.

x2 +1
 
F. Taylor series expansion of f (x) = ln x+1
at x = 0.

G. Taylor series expansion of f (x) = ln(x+1)


(x2 +1)
at x = 0.

I. Taylor series expansion of f (x) = x3 − 3x2 + x + 2 at x = 0.

Exercise: 11.(Taylor Series and Limits)

Compute the limits as x approaches 0 for the following functions:

74
(ESNN)S.Azoug 5.3. SERIES OF EXERCISES ON TAYLOR EXPANSIONS
A. f (x) = x−arcsin(x)
sin3 (x)

B. f (x) = ln(x+1)
tan(x)

C. f (x) = 1
sin2 (x)
− 1
x2

75
(ESNN)S.Azoug CHAPTER 5. LIMIT DEVELOPMENT
5.3.1 Corrections
Exercice 01
1) Signifier que : lim f(x)=0. en x0
2) Signifier que : lim | f (x) |< c
3) Par récurrence : 2n .2 > n(n + 1)
(n + 2)(n + 1)
suffit de montrer que :n(n + 1) >
2
vraie pour n > 2. ensuite déduire par transitivité.
4)xa = 0(xb ) on a que xa = xb .0(1)d’ou: xa−b = 0(1). Condition : a − b > 0.
5)Même raisonnement on trouve : c réel et a − b > 0.
6)Même raisonnement on trouve : a > 0.
Exercice 02
On applique l’équivalence :
sin(x) − sin(0) ∼ (x − 0) cos(0) =⇒ sin(x) ∼ x
tan(x) − tan(0) ∼ (x − 0)(1 + tan2 (0)) =⇒ tan(x) ∼ x
Exercice 03
1) dl3 (π/4) de sin(x) on pose t = x − π/4
sin(x) = sin(t
√ + π/4) = sin(t).cos(π/4) + sin(π/4).cos(t)
(x−π/4)2 3
sin(x) = 2/2 1 + (x − π/4) − 2 − (x−π/4) 6
+ 0(x − π/4)3
= t−t +t t2/3−t /4+o(t )
2 3 4 4
2) ln(x)
x2
= ln(t+1)
(t+1)2 +2.t+1
la division euclidienne donne:
ln(t+1)
(t+1)2
= t − (5/2)t2 + (13/3)t3 /3 − (77/12)t4 /4 + o(t4 )
ln(x)
(x)2
= (x − 1) − (5/2)(x − 1)2 + (13/3)(x − 1)3 /3 − (77/12)(x − 1)4 + o((x − 1)4 )
Exercice 04
1)dl3 (0) de la fonction,
x2 + 1
!
ln = ln(x2 + 1) − ln(x + 1) = −x + (3/2)x2 − (1/3)x3 + o(x3 )
x+1
2)ln(1 + x.sin(x)) = ln(1 + x2 + o(x3 )) = x2 + o(x3 ).
3)cos(ln(x)) = 1 − (1/2)(x − 1)2 + (1/2)(x − 1)3 + o(x − 1)3
4)cos(sh(x)) = 1 − (1/2)x2 + o(x3 )
Exercice 05
1)ln(1 + e sin(x) ) = ln(2) + (1/2)x + (1/8)x2 + o(x3 )
ln(1 + e sin(x) ).sin(x) = ln(2).x + (1/2)x2 + (1/8 − ln(2)/6)x3 + o(x3 )
 2
2) ln(2+x.sin(x))
x+1
= ln(2+sin(1))
2
+ sin(1)+cos(1)
(2+sin(1))
− ln(2+sin(1))
2
(x−1)
2
+( cos(1)−sin(1)/2
2+sin(1)
− ((sin(1)+cos(1))
2(2+sin(1))2
− cos(1)+sin(1)
2(2+sin(1))
+
ln(2+sin(1)) (x−1)2
4
) 2 + o((x − 1)2 )
3) elle n’est pas definit.
√ 2 7.x3
4) sh(x) + 1 = 1 + 2x − x8 + + o(x3 )
48
Exercice 06
1)arcsin(x2 ) = x2 + o(x3 )
3
2)arctan(x) = x − x3 + o(x4 )
3)(arctan(x + 1))0 = 2+2x+x1
2

on fait la division puis on intègre pour obtenir :


arctan(t) = π4 + (1/2)(t − 1) − (1/4)(t − 1)2 + (1/12)(t − 1)3 + o(t − 1)3
Exercice 07

76
(ESNN)S.Azoug 5.3. SERIES OF EXERCISES ON TAYLOR EXPANSIONS
2
e sh(x) − ecos(x) = 1 − e + x + (1 + e) x2 + o(x2 )
2
e sin(x) − ech(x) = 1 − e + x + (1 − e) x2 + o(x2 )
faire la division pour obtenir :
2
1 − e + x + (1 + e) x2 + o(x2 ) e 2
=1+ x + o(x2 )
1 − e + x + (1 − +
2
e) x2 o(x2 ) 1−e

e sh(x) − ecos(x)
!  e 2  e 2
ln sin(x) ch(x)
= ln 1 + x + o(x ) =
2
x + o(x2 )
e −e 1−e 1−e

77
(ESNN)S.Azoug CHAPTER 5. LIMIT DEVELOPMENT

78
Chapter 6
Integrals

6.1 Antiderivatives functions


Definition 6.1.1. Let I be an interval of R. A function f : I −→ R is integrable if there exists a
differentiable function F : I −→ R such that for every x ∈ I, F 0 (x) = f (x). Such a function F is
called a ** antiderivatives** (or indefinite integral) of f .

**Example**
- (sin(x))0 = cos(x), so sin(x) is the antiderivative of cos(x).
- (cos(x))0 = − sin(x), so cos(x) is the antiderivative of − sin(x).
- (sinh(x))0 = cosh(x), so sinh(x) is antiderivative of cosh(x).
- (cosh(x))0 = sinh(x), so cosh(x) is antiderivative of sinh(x).

Figure 6.1: Trigonometric derivatives

**Proposition (Existence)** Let I be an interval of R and f : I −→ R be a continuous function.


Then f is integrable.
**Property** If F is an antiderivative of f , then for any real number c, the function F + c is
also an antiderivative of f .
R
**Notation** The set of antiderivatives of a function f is denoted as f (x)dx.

79
(ESNN)S.Azoug CHAPTER 6. INTEGRALS
Rx
**Remark** If a ∈ I, then F(x) = a f (t)dt is the unique antiderivative that vanishes at a.

6.2 Properties
Let f and g be two integrable functions, and α be a given real number.
- Linearity: Z Z Z
( f (x) + g(x))dx = f (x)dx + g(x)dx
Z Z
α f (x)dx = α f (x)dx

6.3 Computation of antiderivatives


In the following table, integration is assumed to be performed over an interval contained within
the function’s domain of definition.
xn+1 ( f (x))n+1 f 0 (x)
= + =⇒ = + c (n , −1)
R R
xn dx n+1
c (n , −1) ( f (x)) n
dx
R f 0 (x) n+1
dx = ln(x) + c =⇒ dx = ln(| f (x)|) + c
R 1
Rx R f (x)
e x dx = e x + c =⇒ R e f (x) dx = f 0 (x)e f (x) + c
R sin(x)dx = − cos(x) + c =⇒ R f (x) sin( f (x))dx = − cos( f (x)) + c
R
0

cos(x)dx = sin(x) + c =⇒ 0
R f (x)f 0cos( f (x))dx = sin( f (x)) + c
dx = arcsin(x) + c =⇒ dx = arcsin( f (x)) + c
R 1 (x)

2

R 1−x 1−( f (x))2
f 0 (x)
2 dx = arctan(x) + c =⇒ dx = arctan( f (x)) + c
1
R
1+x 1+( f (x))2

R sinh(x)dx = cosh(x) + c =⇒ R f (x) sinh( f (x))dx = cosh( f (x)) + c


R R
0

cosh(x)dx = sinh(x) + c =⇒ f 0 (x) cosh( f (x))dx = sinh( f (x)) + c


**Proposition: Integration by Change of Variable** Let F be an antiderivative of f and g be a
differentiable function. Then the function f (g(x))g0 (x) is integrable, and we have
Z
f (g(x))g0 (x)dx = F(g(x)) + c.

In other words, by setting t = g(x), we obtain dx


dt
= g0 (x), or dt = g(x)dx, and thus
Z Z
f (g(x))g (x)dx =
0
f (t)dt = F(t) + c.

R ln(x)
**Example** Calculate x
dx using a change of variable.
Let t = ln(x), so dt = 1x dx.

t2 ln2 (x)
Z Z
ln(x)
dx = tdt = + c = +c
x 2 2

80
(ESNN)S.Azoug 6.4. DEFINITE INTEGRALS

**Proposition: Integration by Parts**


Z Z
f (x)g (x)dx = f (x)g(x) −
0
f 0 (x)g(x)dx

**Example** Calculate ln(x)dx using integration by parts. If we set g(x) = ln(x) and f (x) =
R

1, then g0 (x) = 1x and f 0 (x) = 0.


Z Z
1
1 · ln(x)dx = x ln(x) − x · dx = x ln(x) − x + c
x

**Antiderivatives of Elementary Functions**

• e x dx = e x + c
R

• = − ln(|1 − x|) + c
R 1
1−x
dx

• sin(x)dx = − cos(x) + c
R

• cos(x)dx = sin(x) + c
R

• tan(x)dx = − ln(| cos(x)|) + c


R

• sinh(x)dx = cosh(x) + c
R

• cosh(x)dx = sinh(x) + c
R

• tanh(x)dx = ln(| cosh(x)|) + c


R

6.4 Definite Integrals


Theorem 6.4.1. Fundamental Theorem of Calculus Let f be a continuous R x function on [a, b]
and F be a primitive of f on [a, b]. Then: 1. The derivative of g(x) = a f (t)dt exists and is
Rx
equal to f (x). 2. a f (t)dt = F(b) − F(a) = [F(x)]ba .

81
(ESNN)S.Azoug CHAPTER 6. INTEGRALS
Rb
**Geometric Interpretation** The definite integral a f (x)dx = F(b) − F(a) corresponds to
the area of the region in the plane between the curve of f and the x-axis, and the two lines with
equations x = a and x = b, as shown in Fig (4.1). **Property**
Let [a, b] be an interval in R with a < b, and let f and g : [a, b] → R be two integrable
functions.
- **Linearity**: Z b Z b Z b
( f (x) + g(x))dx = f (x)dx + g(x)dx
a a a

- **Scalar Multiplication**:
Z b Z b
α f (x)dx = α f (x)dx
a a

for any α ∈ R.
- **Chasles’ Relation**:
Z b Z c Z b
f (x)dx = f (x)dx + f (x)dx
a a c

for any c ∈ [a, b].


- **Order Relation**: If f (x) ≤ g(x) for all x ∈ [a, b], then
Z b Z b
f (x)dx ≤ g(x)dx
a a

Rb
- **Nullity**: If f is continuous on [a, b] and f (x) ≥ 0 for all x ∈ [a, b], then if a
f (x)dx = 0,
then f (x) = 0.
- **Parity**: If f is even on [−a, a] with a ≥ 0, then
Z a Z a
f (x)dx = 2 f (x)dx
−a 0

If f is odd on [−a, a] with a ≥ 0, then


Z a
f (x)dx = 0
−a
.
R2 R π/4 Re
**Application Exercise** Prove that: - x
0 x+1
dx = 2−ln(3) - 0
cos(2x)dx = 1
2
- 1
ln(x)dx =
R4 √
1 - 1 1−x x dx = −1

82
(ESNN)S.Azoug 6.5. SERIES OF EXERCISES ON INTEGRALS
6.5 Series of exercises on integrals
Exercise : 1.(Powers)
Calculate the antiderivatives of the following functions
1. f1 (x) = 5.x4 − 3.x2 + 6.x − 7
√ √ 1
2. f1 (x) = 5. x3 − 3. x.x2 + 6. √
x
2
1 x 3
3. f1 (x) = 5. 2 − 3. √ + 6. √x 3
x x x
Exercise : 2.(trigonometrics)
Calculate the antiderivatives of the following functions:
1. f2 (x) = 5.sin(x) − 3.cos(x) − 7
2. f2 (x) = 2.sin(3x) − 3.cos(2x)
3. f2 (x) = 2.sin2 (x)
4. f2 (x) = 3.cos3 (x)
5. f2 (x) = tan(x)
Exercise : 3.()
Calculate
R sin(ln(x))the following integrals:
1. .dx
R 1+e √x x
2. √ x .dx
R ex
3.R 1+e x .dx

4. √ 1
.dx
x. ln( 1x )
5. e x ln(1 + e x ).dx
R

Calculate the following integrals with changes of variables : (t = ln(x), t = x, t = e x , t =
ln(1/x), t = 1 + e x )
Exercice : 4.
(formulas) Calculate the antiderivatives of the following functions :
1. f4 (x) = (3 + x)7
2. f4 (x) = (4x + 3)1/3
3. f4 (x) = 22x .3 x
4. f4 (x) = x.e x
2

Exercise : 5.
(integrations BP) Calculate the antiderivatives of the following functions:
1. f5 (x) = x.ln(x)
2. f5 (x) = arctan(x)
3. f5 (x) = x.cos(x)
4. f5 (x) = ln2 (x)
5. f5 (x) = cos(x).e x
Exercise : 6.(fractions)
Calculate the antiderivatives of the following functions :
1. f5 (x) = x25x+3 −3x+2
2. f5 (x) = x2 +4x+51

3. f5 (x) = x3 +1 1

4. f5 (x) = 2x−3
x3 +x
Exercise : 7.

83
(ESNN)S.Azoug CHAPTER 6. INTEGRALS
Pour tout entier n , on pose : Z 1 √
In = xn . 1 − x.dx
0
1. en calculant In−1 − In deduce a relation between In − In−1
2. calculate I0
3. Deduce In .

84
Chapter 7
Differential Equations

7.1 Introduction
Differential equations are mathematical equations that involve one or more functions and their
derivatives. These equations describe how a function changes as its input (or inputs) change.
Differential equations are used to model a wide range of phenomena in various fields such as
physics, engineering, biology, and economics.
There are different types of differential equations, including ordinary differential equations
(ODEs) and partial differential equations (PDEs). ODEs involve functions of a single variable,
whereas PDEs involve functions of multiple variables.
Differential equations play a crucial role in understanding dynamic systems and predicting their
behavior over time. They are often used to represent the relationships between different vari-
ables in a system and are fundamental in areas such as classical mechanics, electromagnetism,
fluid dynamics, and heat transfer.
Solving differential equations can be a challenging task, as it requires finding a function that
satisfies the given equation. Depending on the complexity of the equation, solutions can be
analytical (expressed in terms of elementary functions) or numerical (approximated using nu-
merical methods and computational techniques).
Understanding and solving differential equations are essential skills for scientists, engineers,
and researchers working in various fields, as they provide valuable insights into the behavior of
dynamic systems and help in making predictions and decisions based on mathematical models.

As the general form of an ordinary differential equation is an equation establishing a rela-


tionship between the variable x and the unknown y = f(x), and y0 , y(2) , ..., y(n) are the successive
derivatives of y. The highest order, n, is the order of the differential equation, as follows:

F(x, y, y0 , ..., y(n) ) = 0.

Here’s a physical model represented by a differential equation:


Consider a water reservoir with an outflow pipe. The water height in the reservoir can be
represented by the function h(t), where t is time. The equation for the water outflow rate from
the reservoir can be described by Torricelli’s law:

85
(ESNN)S.Azoug CHAPTER 7. DIFFERENTIAL EQUATIONS

2gh(t) = Q(t)
p

where A is the cross-sectional area of the outflow pipe, g is the acceleration due to gravity, and
Q(t) is the water flow rate leaving the reservoir at a given time t.
Using this equation, we can establish a differential equation by introducing the change in water
height over time dh
dt
. Assuming that the rate of change of height dh
dt
is proportional to the current
height difference h(t) and a target height hc , we get the following differential equation:

dh
2gh(t) = k · (hc − h(t)) ·
p

dt

where k is a proportionality constant.


This differential equation models the behavior of water height in the reservoir over time, taking
into account the outflow rate and the height difference between the current height and the target
height.

The Navier-Stokes equations are a set of partial differential equations that describe the mo-
tion of fluid substances, such as liquids and gases. They are named after the French engineer
and physicist Claude-Louis Navier and the Irish mathematician and physicist George Gabriel
Stokes. The Navier-Stokes equations can be written in their general form as follows:

∂v
!
ρ + v · ∇v = −∇p + µ∇2 v + f
∂t

where: - ρ is the density of the fluid, - v is the velocity vector field of the fluid as a function
of spatial coordinates (x, y, z) and time t, - p is the pressure of the fluid as a function of spatial
coordinates (x, y, z) and time t, - µ is the dynamic viscosity of the fluid, - ∇ represents the
gradient operator, - f represents any external forces acting on the fluid, such as gravitational
forces.
The Navier-Stokes equations are fundamental in fluid mechanics and are used to study a wide
range of phenomena, including the flow of air around an airplane wing, the motion of ocean
currents, the behavior of blood flow in arteries, and many other fluid dynamics problems.
Understanding the solutions to the Navier-Stokes equations is crucial in various engineering
applications, including aerospace, automotive design, weather prediction, and the design of
pipelines and water treatment systems. However, solving these equations analytically can be
extremely challenging, especially for complex geometries and turbulent flows. As a result,
numerical methods and computational simulations are often employed to approximate solutions
for practical engineering purposes.
”Example 01: y + 2x y0 + y00 = 0 ...(*) is a second-order equation.

sin(x) cos(x)
y = C1 + C2
x x

86
(ESNN)S.Azoug 7.2. FIRST-ORDER DIFFERENTIAL EQUATIONS
is the general solution where C1 and C2 are arbitrary constants. To verify, calculate y0 and y00
and substitute them into equation (*). Example 02: y0 − 3y = 0 ...(**) is a first-order equation.
y = Ce3x
is the general solution where C is an arbitrary constant. To verify, calculate y0 and substitute it
into equation (**). Remarks: The general solution of a differential equation of order n depends
on n arbitrary constants. By assigning specific values to these constants, we obtain a particular
solution. Example 03: Taking C = 2, we get y = 2e3x as a particular solution of (**).

7.2 First-order Differential Equations

7.2.1 Separable Differential Equations


General Form:”

y0 = f (x).g(y)

**Translation:** Resolution

• We write y0 = dx
dy
= f (x)g(y) ⇐⇒ dy
g(y)
= f (x)dx

• We integrate g(y) = f (x)dx


R dy R

Example Solve the following differential equation:


y0 = y.(x2 + x + 1)

• We write y0 = dx
dy
= y.(x2 + x + 1) ⇐⇒ dy
y
= (x2 + x + 1)dx

• We integrate dyy = (x2 + x + 1)dx


R R

 3
x2

 x 
 + +x+c

3 2

• y=e

Note Sometimes it is not possible to express the solution y as a function of x. The solution
then appears in implicit form.

Example Solve the following differential equation:


yy0
+ ey = 0
x
• We write y.e−y dy + x.dx = 0 =⇒ y.e−y dy + x.dx = 0
R R

x2
• We integrate and obtain: − (y + 1).e−y = c
3

87
(ESNN)S.Azoug CHAPTER 7. DIFFERENTIAL EQUATIONS
7.2.2 First-Order Homogeneous Differential Equations

General Form

y0 = f (x, y)

**Translation:**
f is a homogeneous function of degree zero, meaning: for any non-zero α, f (αx, αy) = f (x, y).
In this case, it can always be shown that it is possible to express y0 in terms of the ratio u = yx .

Example Solve the following differential equation:

3yx
y0 =
x2+ y2

• We write f (αx, αy) = f (x, y)

• y0 = f (x, αx) = 3u
1+u2

• dy
dx
= y0 = x du
dx
+u

• Thus, we obtain a separable variable equation: du


f (u)−u
= dx
x

7.2.3 First-Order Differential Equations

First-Order Linear Homogeneous Differential Equations

General Form

y0 + f (x)y = 0

Solution

• We write y0 + f (x)y = 0 ⇐⇒ =
R dy
R
y
− f (x)dx
R
• We integrate y = C.e− f (x)dx

Example Solve the following differential equation:

y0 + e x y = 0

88
(ESNN)S.Azoug 7.2. FIRST-ORDER DIFFERENTIAL EQUATIONS
First-Order Non-Homogeneous Linear Differential Equations

General Form

y0 + f (x)y = g(x)

Solution

• We seek a solution to the homogeneous equation: y00 + f (x)y0 = 0

• Then we seek a particular solution of: y0p + f (x)y p = g(x)

• The general solution: y = y0 + y p

**Translation:**

Example Solve the following differential equation:

(x2 − 1)y0 − (2x + 1)y = 1

Knowing that it has a particular solution in the form of a second-degree polynomial.


Solution:
√ By identification, we find y p = (2/3)x2 − (2/3)x − 1/3 and the solution y0 = C(x −
1) x2 − 1. The general solution is obtained by taking the sum of these solutions.

Method ofR Variation of Constants We solve the associated homogeneous equation to find:
y0 = C.e− f (x)dx . Then
R we solve the complete equation, considering the constant as a function
of x: y0 = C(x).e
R
− f (x)dx
. So, we look
R for C(x). R
y = C (x).e =⇒ C(x) = g(x).e f (x)dx + K
R
0 0 − f (x)dx − f (x)dx
− f (x).C(x).e

Example Solve the following differential equation:

y0 + y = e2x

• We look for a solution to the homogeneous equation: y00 + y0 = 0, which gives y = C.e−x

• Then we seek a particular solution of: y0p + y p = e x

e2x
• We find C(x) = e3x and the solution: y = 3
+ K.e−x

89
(ESNN)S.Azoug CHAPTER 7. DIFFERENTIAL EQUATIONS
7.2.4 Bernoulli’s Differential Equation
General Form

y0 + f (x)y = g(x)ym

For a non-linear differential equation, if m = 0 or m = 1, it reduces to the previous case.


Solution

• Let z = y1−m =⇒ z0 = (1 − m)y−m y0


y0
• Dividing by ym , we get ym
+ f (x) yym = g(x)
z0
• 1−m
+ f (x)z = g(x), which is a first-order linear equation.

Example Solve the following differential equation:


2xy0 = 10x3 y5 + y

Solution Rewrite the form as: y0 − 2x1 y = 5x2 y5 .


We deduce that g(x) = 5x2 , f (x) = − 2x1 , and z = y−4 .
y0
• Dividing by y5 , we get − 1
2x.y4
= 5x2
y5
• Hence, z0 + 2x z = −20x2 (a linear equation)
• Solve z0 + 2x z = 0 to get z = c
x2

• Variation of constants gives: z p = −4.x3


• The general solution is: z = −4.x3 + c
x2

• As z = y−4 , y = 1
(−4.x3 + c 1/4
)
x2

**Translation:**

7.2.5 Riccati’s Differential Equation


General Form

y0 = A(x)y2 + B(x)y + C(x)

For a non-linear differential equation, if m = 0 or m = 1, it reduces to the previous case.


Solution
The equation can only be solved if a particular solution y p is known. So, one must begin by
searching for a solution of a specific form of the equation.

• We make the following change: z = 1


y−y p

• We find z0 = −(B(x) + 2y p A(x))z − A(x), which is a first-order linear equation.

90
7.3. SECOND-ORDER HOMOGENEOUS DIFFERENTIAL EQUATIONS WITH
(ESNN)S.Azoug CONSTANT COEFFICIENTS
Example Solve the following differential equation:
xy0 − y2 + (2x + 1)y = x2 + 2x
Solution

1. Clearly y p = x is a particular solution (to find one, identification with y = ax + b is


sufficient)
z0
2. We let y = x + 1z , hence y0 = 1 − z2

3. The equation becomes z0 = z


x
− 1z , a first-order equation.

4. The solution is z = cx + 1, therefore: y = x + 1


cx+1

Exercise Solve the following differential equation:


xy0 = y2 + y − 2

c+2x3
Solution The solution is: y = c−x3

7.3 Second-Order Homogeneous Differential Equations with


Constant Coefficients
General Form

y + by0 + cy = 0
00

Solution:
To this differential equation, we associate a so-called ”characteristic equation” given by:
r2 + ar + b = 0
We calculate the discriminant ∆ = b2 − 4ac

• If ∆ = b2 − 4c > 0, this equation has two roots:



−b − ∆
r1 =
2
, √
−b + ∆
r2 =
2
The differential equation then has the solution:
y = C1 er1 x + C2 er2 x
where C1 and C2 are arbitrary constants.

91
(ESNN)S.Azoug CHAPTER 7. DIFFERENTIAL EQUATIONS
• If ∆ = b − 4c = 0, this equation has a double root:
2

−b
r0 =
2
The differential equation then has the solution:

y = (C1 x + C2 )er0 x

where C1 and C2 are arbitrary constants.

• If ∆ = b2 − 4c < 0, this equation has two complex conjugate roots:

r1 = α − iβ

,
r2 = α + iβ
The differential equation then has the solution:

y = eαx (C1 cos(βx) + C2 sin(βx))

**Translation:**

Examples

• y − 3y0 + 2y = 0
00

**Solution:** The characteristic equation is

r2 − 3r + 2 = 0

The general solution is


y = C1 e2x + C2 e x

• y − 2y0 + y = 0
00

**Solution:** The characteristic equation is

r2 − 2r + 1 = 0

The general solution is


y = (C1 x + C2 )e x

• y + y0 + y = 0
00

**Solution:** The characteristic equation is

r2 + r + 1 = 0

The general solution is


√ √
y1 = e(−1/2)x (c1 cos( 3x) + c2 sin( 3x))

92
7.4. SECOND-ORDER NON-HOMOGENEOUS DIFFERENTIAL EQUATIONS WITH
(ESNN)S.Azoug CONSTANT COEFFICIENTS
7.4 Second-Order Non-Homogeneous Differential Equations
with Constant Coefficients
General Form
y + by0 + cy = f (x)
00

Solution:
The general solution y is the sum of yh , solution of y” + by0 + cy = 0, and a particular solution
y p of the equation associated with this non-homogeneous equation y” + by0 + cy = f (x), thus:
y = y p + y0
To find a particular solution, the ”method of variation of constants” is used.

The Method of Variation of Constants We look for a particular solution in the following
form:
y p = C1 y1 + C2 y2
with:
• If ∆ = b2 − 4c > 0, take: y1 = er1 x and y2 = er2 x
• If ∆ = b2 − 4c = 0, take: y1 = xer0 x and y2 = er0 x
• If ∆ = b2 − 4c < 0, take:
y1 = eαx cos(βx)
and
y2 = eαx sin(βx)
The method consists of considering C1 (x) and C2 (x) as functions and then solving the following
system of equations:

C1 y1 + C20 y2 = 0
( 0

C10 y01 + C20 y02 = f (x)

Examples Solve
00
y − 3y0 + 2y = x
Using the method of variation of constants, we get the system of equations to solve:

 C1 e + C2 e = 0
 0 x 0 2x


C10 e x + 2C20 e2x = x

After solving the system, the particular solution y p can be obtained.


• on obtient donc C1 = (x + 1)e x et C2 = −( 2x + 14 )e−2x
• la solution particulière y p = x
2
+ 3
4

• la solution générale y = C1 .e x + C2 .e2x + y p

93
(ESNN)S.Azoug CHAPTER 7. DIFFERENTIAL EQUATIONS
7.5 Série d’exercices sur les équations différentielles
Exercice n 01(variables separées)

Trouver une equation differentielle qui admet pour solution générale la fonction suivante : y =
C.e−x

Exercice n 02(variables separées)

Résoudre les équations suivantes:

• xy’=y

• y’=y+1
1
• y0 = y(1 − )
x2
• 1 + xy0 = ey

• (x2 + 1)y0 = 2xy

• xyy0 = y2 + 1

Exercice n 02(Equations dif.Lineaires 1er ordre Homogènes)

Intégrer les equations suivantes:

• y0 + y = 0

• (1 + x)y0 + y = 0

• x3 y0 − x2 y = 0

• (1 + x2 )y0 + y = 0

• xy0 + y = 0

• (1 + x)y0 + y = 0

Exercice n 02(Equations dif.Lineaires 1er ordre non Homogènes)

Intégrer les equations suivantes:

• xy0 + y = cos(x)

• (1 + x)y0 + y = (1 + x) sin(x)

• x3 y0 − x2 y = 1

94
(ESNN)S.Azoug 7.5. SÉRIE D’EXERCICES SUR LES ÉQUATIONS DIFFÉRENTIELLES
• (1 + x)y0 + y = (1 + x) sin(x)

• xy0 + y = cos(x)

• (1 + x)y0 + y = (1 + x) sin(x)

Exercice n 03Equations De Bernoulli

Intégrer les equations suivantes:

• y0 − y
x
= −x.y2

• x.y0 − y = y2 . log(x)

• y0 = xy(1 − y2 )

• y0 = xy(x2 .y2 − 1)

• y0 + 2xy = 2xy2

• y0 − 2ye x = 2 ye x

Exercice n 04Equations De Riccati

• y0 = y2 − 2xy + x2 + 1

• y0 = y2 − 2ye x + e2x + e x

• xy0 − y2 + (2x + 1)y = x2 + 2x

• y0 .e−x + y2 − 2ye x + sin2 (x) − cos(x) = 0

Exercice n 05Équations 2nd Degré Homogènes

Intégrer les equations suivantes:

• y − 2y0 + y = 0
00

• y +y=0
00

• 3y + y0 − 4y = 0
00

• 3y + y0 − 4y = 0
00

95
(ESNN)S.Azoug CHAPTER 7. DIFFERENTIAL EQUATIONS
7.5.1 Correction des exercices sur les équations différentielles
Exercice n 01

On calcul sa dérivée et on fait la somme pour obtenir: y + y0 = 0.

Exercice n 02(variables separées)

Résolutions des equations différentielles:

• xy0 = y =⇒ y = cx

• y0 = y + 1 =⇒ y = c.e x − 1

• y0 + y = 0 =⇒ y = c.e−x

• (1 + x)y0 + y = 0 =⇒ y = c
x

• x3 y0 − x2 y = 0 =⇒ y = cx

• (1 + x2 )y0 + y = 0 =⇒ y = c.e−arctan(x)

• x.y0 + y = 0 =⇒ y = c
x

• x.y0 + y = cos(x) =⇒ y = (c + sin(x))/x

• x.y0 + y = e x =⇒ y = (c + e x )/x

96
Chapter 8
Functions of Two Variables

8.1 Generalities
Throughout this chapter, let P denote a subset of R2 , meaning a plane excluding a point, a half-
plane, a disc, a rectangle, an ellipse, a circle, etc.

8.1.1 Topology Vocabulary in R2


R2 is equipped with pthe standard distance defined as follows: for any points A(xA , yA ) and
B(xB , yB ), d(A; B) = (xA − xB )2 + (yA − yB )2 .
Definition 1. Open Set in R2
We say that P is an open set in R2 if and only if every point in P is the center of an open disk
that is completely contained within P, meaning:

∀M0 (x0 , y0 ) ∈ P, ∃r > 0, d(M, M0 ) < r =⇒ M ∈ P

Definition 2. Closed Set in R2


We say that P is a closed set in R2 if and only if its complement is an open set.
Definition 3. Compact Set in R2
We say that P is a compact set in R2 if and only if it is a closed and bounded set.
Definition 4. Boundary of R2
The boundary of P is the set of points M0 such that for every open disk centered at M0 , there is
at least one point inside the disk belonging to P and one point outside the disk not belonging to
P.
Examples Draw the following sets specifying their natures and boundaries:
n o
E1 = (x, y) ∈ R2 : d((0, 0); (x, y)) ≤ 1
n o
E2 = (x, y) ∈ R2 : d((1, 0); (x, y))<1
n o
E3 = (x, y) ∈ R2 : d((0, 0); (x, y)) ≥ 1

97
(ESNN)S.Azoug CHAPTER 8. FUNCTIONS OF TWO VARIABLES

8.2 Functions with Two Variables


Definition: A function of two variables is defined by the following relation:

f : P →R2 (8.1)
(x, y) →z = f (x, y) (8.2)

Its graphical representation is a surface in R3 .

8.2.1 Partial Functions in R2

Let f : P → R2 and M0 (x0 , y0 ) ∈ P. The partial functions associated with f at M0 are functions
from R2 to R2 defined as follows: Let fy0 : x → f (x, y0 )
f x0 : y → f (x0 , y)
Example: Determine the partial functions of f (x, y) = e−x −y at the point (0, 0).
2 2

8.2.2 Representation by Level Curves

To obtain partial functions, we consider vertical cross-sections of the graph of a function of two
variables. Similarly, considering horizontal cross-sections results in plane curves, called level
curves.

Definition 10 (Level Curves) Let k ∈ R and f be a function from R2 to R. The level curve
k of the function f is the projection onto the plane z = 0 of the intersection of the surface
representing f with the horizontal plane z = k, i.e., the set (x, y) ∈ D : f (x, y) = k. In practice,
different level curves are simultaneously represented to visualize the graph’s behavior. This
representation is similar to geographical maps where the level corresponds to altitude. Level
curves of a function f (x, y) provide a geometric representation of f in the plane, whereas its
graph provides one in space.

98
(ESNN)S.Azoug 8.2. FUNCTIONS WITH TWO VARIABLES

Figure 8.1: •

Figure 8.2: lignes de niveau( Imedghassen - Batna )

8.2.3 Limits and Continuity

Definitions and Properties Definitions: Let f be a function defined on a subset V of R2 ,


taking real values, and M0 a point in R2 . We say that lim f (M) = l if:
M→M0

∀ε>0, ∃α>0 s.t. ∀M ∈ V, d(M0 , M) ≤ α =⇒ | f (M) − l| ≤ ε

This means: for any non-zero distance ε (however small), there always exists an open disk
centered at M0 with radius α, such that for any point M inside the disk and in the domain of
definition, the difference between f (M) and l is smaller than ε.
Similarly, we say that lim f (M) = +∞ if and only if:
M→M0

∀A ∈ R, ∃α>0 s.t. ∀M ∈ V, d(M0 , M) ≤ α =⇒ f (M) ≥ A

And we say that lim f (M) = −∞ if and only if:


M→M0

∀A ∈ R, ∃α>0 s.t. ∀M ∈ V, d(M0 , M) ≤ α =⇒ f (M) ≤ A

99
(ESNN)S.Azoug CHAPTER 8. FUNCTIONS OF TWO VARIABLES
Example
Compute lim f (M) with:
M→O
f1 (x, y) = sin(x2 + y2 ),
f2 (x, y) = ln(x2 + y2 )
All the theorems applicable to functions of one variable are applicable to functions of two
variables as well. In particular, the Squeeze Theorem, which states here:
Theorem Let f :→ R and g :→ R such that for all (x, y) in the neighborhood of (x0 , y0 ),
| f (x, y)| ≤ g(x, y), then: if lim g(x, y) = 0, then lim f (x, y) = 0
(x,y)→(x0 ,y0 ) (x,y)→(x0 ,y0 )
Definition. We say that f is continuous at (x0 , y0 ) if and only if:

lim f (M) = f (M0 )


M→M0

8.3 Practical Computation of Limits


Let M0 be a point belonging to the boundary of the domain of definition and not belonging to
the domain of definition.

8.3.1 The Function Has a Limit at M0


When the calculation of a limit is not straightforward, majorization techniques are often used,
or polar coordinates are employed.
x.y3
Example Consider the function defined by f (x, y) = x2 +y2

• Show that x.y ≤ 12 (x2 + y2 )

2
• Deduce that | f (x, y)| ≤| y2 |

• Find the limit of f at (0, 0).

8.3.2 The Function Does Not Have a Limit at M0


Path Rules Let u and v be two continuous functions such that lim u(x) = y0 and lim v(y) =
x→x0 y→y0
x0 . If f has a limit l at (x0 , y0 ) then lim f (x, u(x)) = l and lim f (v(y), y) = l.
x→x0 y→y0

Remark If f has equal limits along one or more paths, it does not necessarily mean that f
has a limit.

Proposition If the limits along two different paths are not equal, it follows that f does not
have a limit at (x0 , y0 ).
(This proposition will be used to show that f does not have a limit.)

100
(ESNN)S.Azoug 8.4. DIFFERENTIAL CALCULUS
Example Let f be a function defined by: f (x, y) = x.y
x2 +y2
- Show that f does not have a limit at (0, 0).

8.4 Differential Calculus


Let U be an open subset of R2 and f a function from U to R.
We say that f has partial derivatives at M0 (x0 , y0 )
if lim f (x0 +t,y0 )−
t
f (x0 ,y0 )
and lim f (x0 ,y0 +t)−
t
f (x0 ,y0 )
are finite.
t→0 t→0
We then denote:
f (x0 + t, y0 ) − f (x0 , y0 ) ∂ f
lim = (x0 , y0 )
t→0 t ∂x
f (x0 + t, y0 ) − f (x0 , y0 ) ∂ f
lim = (x0 , y0 )
t→0 t ∂y
It is observed that the partial derivatives are the derivatives of the partial functions f x0 and fy0 .
Example Calculate the partial derivatives of f (x, y) = (x + y).e−x −y .
2 2

8.4.1 Functions of Class C 1


Consider a function f from U to R having partial derivatives at every point in U. We say that f
is of class C 1 on U if and only if the partial derivatives of f are continuous on U.

Notation The set of C 1 functions on U is denoted by C 1 (U).

8.4.2 Differentiability
Differentiable Function If there exist real numbers m and p, and a disk D centered at (x0 , y0 )
such that for every (h, k) ∈ D:

f (x0 + h, y0 + k) = f (x0 , y0 ) + m.h + p.k + k(h, k)k.Φ(x0 + h, y0 + k)

with lim Φ(x0 + h, y0 + k) = 0, then we say that f is differentiable at (x0 , y0 ). We have


(h,k)→(0,0)
∂f ∂f
m= (x , y )
∂x 0 0
and p = (x , y )
∂y 0 0

Remark: If f has partial derivatives at (x0 , y0 ), f is not necessarily differentiable at (x0 , y0 ).

x.y2
Exercise: Let f be a function defined by f (x, y) = x2 +y2
if (x, y) , (0, 0) and f (0, 0) = 0.

• Show that f has two partial derivatives at (0, 0), and compute its partial derivatives.

101
(ESNN)S.Azoug CHAPTER 8. FUNCTIONS OF TWO VARIABLES
• If f is differentiable at (0, 0), find the function Φ
.

• Calculate Φ(x, x) for x ¿ 0.

• Conclude.

Proposition If f is C 1 , then f is differentiable.

Remark This means if f is not differentiable, it is not of class C 1 .

Differential Map Let f be a differentiable function at (x0 , y0 ). The map d f (M0 ) : (h, k) →
h. ∂∂xf (M0 ) + k. ∂∂yf (M0 ) is a linear map from R2 to R. It is called the differential of f at M0 .

8.4.3 First Order Taylor Expansion

Definition The first order Taylor expansion of f at M0 is given by the formula:

−−→
f (x0 + h, y0 + k) = f (x0 , y0 ) + h. ∂∂xf (x0 , y0 ) + k. ∂∂yf (x0 , y0 ) + o(kh, kk)

Example 1 Let S = x.y be the surface of a rectangle using dl(10, 2) with dx = 0.1, dy = 0.01.
Determine the variation of the surface.

Example 2 Write dl1 (π, 1) of f (x, y) = x3 + y3 . cos(x).

8.4.4 Tangent Plane to a Surface

The tangent plane to a surface (similar to the tangent to a curve) at a point (x0 , y0 , f (x0 , y0 )) on
the surface defined by z = f (x, y) is given by:

z = f (x0 , y0 ) + (x − x0 ). ∂∂xf (x0 , y0 ) + (y − y0 ). ∂∂yf (x0 , y0 )

8.4.5 Gradient

Definition Let f be C 1 on U. At every point M0 , there exists a unique vector called the
gradient of f at M0 denoted as grad f (M0 ) with coordinates ∂x (M0 ), ∂∂yf (M0 ) .
∂f

Example Calculate the gradient of f (x, y) = x2 − xy + x − 2.

102
(ESNN)S.Azoug 8.4. DIFFERENTIAL CALCULUS
8.4.6 Jacobian Matrix
Definition 1. The Jacobian matrix of a function f from R2 to R at M0 (x0 , y0 ) is the matrix
∂f ∂f
!
J( f )(M0 ) = (M0 ), (M0 )
∂x ∂y

Definition 2. The Jacobian matrix of a function Φ from R2 to R2 at M0 (x0 , y0 ) is the matrix


∂f ∂f
!
J( f )(M0 ) = (M0 ), (M0 )
∂x ∂y

 ∂Φ1 (M ) ∂Φ1 (M )


 
 ∂x 0 0 
∂y
J(Φ)(M0 ) = 

 ∂Φ2 ∂Φ2


(M0 ) (M0 )
∂x ∂y
where Φ = (Φ1 , Φ2 ).

Definition 3. Let f be defined on U into R of class C 1 , and Φ from V ⊂ R2 into U, of class


C 1 on V. Then g = f oΦ is of class C 1 on V for every M ∈ V, and we have:

J(g)(M) = J( f )(Φ(M)) · J(Φ)(M)

J(g)(M)=( ∂∂xf (M0 ), ∂∂yf (M0 )).

∂Φ ∂Φ1
 
!  1 (x, y) (x, y)
∂f ∂f  ∂x ∂y
J(g)(M0 ) = (Φ1 (x, y), Φ2 (x, y)), (Φ1 (x, y), Φ2 (x, y)) × 
 
∂x ∂y  ∂Φ2 ∂Φ2

(x, y) (x, y)

∂x ∂y

∂g ∂f ∂Φ1 ∂f ∂Φ2
• ∂x
= ∂x
· ∂x
+ ∂y
· ∂x

∂g ∂f ∂Φ1 ∂f ∂Φ2
• ∂y
= ∂x
· ∂y
+ ∂y
· ∂y

Remark Jacobians are used to make variable changes in integrals, such as changing from
Cartesian coordinates to polar coordinates.

Definition of Jacobian The Jacobian of Φ is the determinant of the Jacobian matrix J(Φ)(M).

Example Φ : (r, θ) → (r. cos(θ), r. sin(θ))

cos(θ) sin(θ)
det J(Φ)(M) = =r
−r. sin(θ) r. cos(θ)

103
(ESNN)S.Azoug CHAPTER 8. FUNCTIONS OF TWO VARIABLES
8.5 Second Partial Derivatives
∂f
Let f be a function of class C 1 on an open set U in R2 . If the first partial derivatives ∂x
and ∂∂yf
∂2 f ∂2 f ∂2 f ∂2 f
are of class C 1 on U, we say that f is of class C 2 on U. We denote , ,
∂x2 ∂y2 ∂y∂x
, and ∂x∂y
as the
second partial derivatives of f .

Theorem 8.5.1. (Schwarz’s Theorem) Let f be a function of class C 2 on an open set U, then

∂2 f ∂2 f
∂y∂x
(a, b) = ∂x∂y
(a, b)

Counterexample However, without the continuity assumption at the point (a, b), the result
of the theorem becomes false. For example, if

xy(x2 − y2 )
f (x, y) = for (x, y) , (0, 0)
(x2 + y2 )
then the second-order partial derivatives of f at (0, 0) exist, but

∂2 f
(0, 0) = 1
∂x∂y

∂2 f
(0, 0) = −1
∂y∂x
This counterexample is due to Peano.

8.6 Extrema
• We say that f has a local or relative minimum at (a, b) in R2 if f (x, y) ≥ f (a, b) for all
(x, y) in the neighborhood of (a, b).

• We say that f has a global or absolute minimum at (a, b) in R2 if f (x, y) ≥ f (a, b) for all
(x, y) in R2 .

• We say that f has a local or relative maximum at (a, b) in R2 if f (x, y) ≤ f (a, b) for all
(x, y) in the neighborhood of (a, b).

• We say that f has a global or absolute maximum at (a, b) in R2 if f (x, y) ≤ f (a, b) for all
(x, y) in R2 .

104
(ESNN)S.Azoug 8.6. EXTREMA
1 2
Definition Let f be a C function on an open set U in R . We say that M ∈ U is a critical
point of f if:
∂ f
(x, y) = 0


 ∂x



 ∂f
 (x, y) = 0



∂y

Example: Determine the two critical points of f (x, y) = (x + y)e−x −y


2 2

Theorem 8.6.1. Let f be a C 1 function on an open set U in R2 . M(a, b) is a critical point of f


if ∂∂xf (a, b) = 0. and ∂∂yf (a, b) = 0. or if ∂∂xf (a, b) or ∂∂yf (a, b) does not exist.

∂2 f ∂2 f ∂2 f
Monge’s Notation: We denote r = ∂x2
, t= ∂y2
, s= ∂y∂x
. If M(a, b) is a critical point of f
and s2 − r.t , 0, then:

• If s2 − r.t < 0. and r > 0, f has a local or relative minimum at (a, b, f (a, b)).

• If s2 − r.t < 0. and r < 0, f has a local or relative maximum at (a, b, f (a, b)).

• If s2 − r.t > 0. and r , 0, f has a saddle point at (a, b, f (a, b)).

Remark: In the case where s2 − r.t = 0. or when r = 0 or when the partial derivatives ∂∂xf (a, b)
or ∂∂yf (a, b) do not exist, the above test is inconclusive. In such cases, one must examine the
behavior of f in the neighborhood of (a, b) or its graph.

105
(ESNN)S.Azoug CHAPTER 8. FUNCTIONS OF TWO VARIABLES
Exercise Determine the nature of the critical points of the function: f (x, y) = x2 −4xy+y3 +4y

8.7 Lagrange Multipliers


In many problems, when we are looking for the extrema of a function, we add a condition
related to the nature of the problem itself (for example). This condition is g(x, y) = 0, and we
look for the extrema of f with this constraint.

Theorem 8.7.1. (Lagrange) Points at which a function f is of class C 1 on an open set U in


R2 and has an extreme value under the constraint g(x, y) = 0 are given by the points (x,y)
determined by the first two coordinates of the solutions (x, y, λ) of the following system:

∂f ∂g

(x, y) = λ. (x, y)


∂x ∂x





∂ f


∂g
 (x, y) = λ. (x, y)
∂y ∂y






 g(x, y) = 0

The number λ is called the Lagrange multiplier.

Exercise Consider the function f (x, y) = xy. Determine the maxima of f within the ellipse
defined by the equation: 4x2 + y2 = 4.

8.8 Cobb-Douglas Function


Consider the following production model in which an output Y is produced using two inputs:
capital K and labor L, according to the production function:

Y = f (K, L) = K α .L1−α

with 0 < α < 1.

106
(ESNN)S.Azoug 8.8. COBB-DOUGLAS FUNCTION
Calculation of Marginal Productivities, Relations with Average Productivities:

∂f
• ∂K
= α.( KL )1−α = α. KY

∂f
• ∂L
= (1 − α).( KL )α = (1 − α). YL

The Cobb-Douglas Production Function The Cobb-Douglas production function is of the


form:
Q = A.K α .Lβ
where A is a coefficient with characteristic dimensions of the economy and the units of mea-
surement used;

1. K = amount of capital used;

2. L = amount of labor used;

3. α = share of production that remunerates K;

4. β = share of production that remunerates L;


with α + β = 1.

Figure 8.3: Cobb-Douglas Surface α = 0.5, β = 0.5

The Function is Homogeneous of Degree 1 The returns are assumed to be constant. If we


multiply K and L by t:

Q(tK, tL) = A.(t.K)α (tL)β = A.tα+β .K α .Lβ = t.AK α .Lβ = t.Q

107
(ESNN)S.Azoug CHAPTER 8. FUNCTIONS OF TWO VARIABLES
Partial Elasticities of Production with Respect to Factors

1. eQ/K = .
dQ K
dK Q
= Q0K . QK = α

2. eQ/L = β = 1 − α if α + β = 1.

108
(ESNN)S.Azoug 8.9. SERIES OF EXERCISES ON FUNCTIONS OF TWO VARIABLES
8.9 Series of exercises on functions of two variables
Exercise 01
Graphically represent the following sets of points on a Cartesian plane:
1. D1 = {(x, y) ∈ R2 /x ≥ 0 and y ≥ x}
2. D2 = {(x, y) ∈ R2 /x ≥ 1 and y + x − 3 6 0}
3. D3 = {(x, y) ∈ R2 /x ≥ 0 and y2 + x2 ≥ 1}
4. D4 = {(x, y) ∈ R2 /y ≥ 0 and y2 + x2 ≥ 1}
5. D5 = {(x, y) ∈ R2 /x ≥ 0, y ≥ 0, 3y + x − 12 6 0, y + 3x − 12 6 0}

Exercise 2
Determine the level curves of the following functions:
1. f (x, y) = x + y − 12
2. f (x, y) = x2 y
3. f (x, y) = x2 + y2
4. f (x, y) = xy2
5. f (x, y) = ey−x
2

6. f (x, y) = y − cos(x)

Exercise 3
Graphically represent the domain of definition for the following functions:
3 2x
1. f (x, y) = x y+y
x+y
2. f (x, y) = x2 +y
1
2

3. f (x, y) = xy
1

4. f (x, y) = √5 − x2 − y2
p

4−x2 −y2
5. f (x, y) = √ 2 2
p x +y −1
6. f (x, y) = ln(xy)

Exercise 4
Compute thep limits if they exist or show that they do not exist:
1. lim x2 + xy + y2
(x,y)→(0,0)
1
2. lim
(x,y)→(0,0) x +y
2 2

xy
3. lim √
(x,y)→(0,0) x2 +y2
1
4. lim
(x,y)→(1,1) x−y
3
5. lim (x−1)y 2 +y2
(x,y)→(1,0)
6. lim (x + y) sin( x2 +y
1
2)
(x,y)→(0,0)
x2 +y2
7. lim
(x,y)→(0,0) x +y
4 4

x+y
8. lim
(x,y)→(∞,∞) x +y
2 2

y sin(x+y)
9. lim x2 −1
(x,y)→(1,2)

109
(ESNN)S.Azoug CHAPTER 8. FUNCTIONS OF TWO VARIABLES
10. lim x2 +yxy−2y
2 −4x+4
(x,y)→(2,0)

Exercise 5:
Study the continuity of the following functions:
1.
x.y2

 √ x2 +y2 if (x, y) , (0, 0)


f (x, y) = 

if (x, y) = (0, 0)

0

2. 
x
y2 sin( y ) if y , 0

g(x, y) = 

0
 if y = 0
3.  x3 +y3

 x2 +y2 if (x, y) , (0, 0)
h(x, y) = 

0
 if (x, y) = (0, 0)
4. 
 21 .x2 + y2 − 1 if x2 + y2 > 1.

k(x, y) = 

− 1 x2
 otherwise
2

Exercise 6:
Graphically represent the domain of definition along with the first partial derivatives:
1. f (x, y) = y5 − 3xy.
2. f (x, y) = x2 + 3xy2 − 6y5 .
3. f (x, y) = x. cos(e x.y ).
4. f (x, y) = yx .
5. f (x, y) = xy
6. f (x, y) = x2ln(x) .
√+y −9
2

7. f (x, y) = x.y.
8. f (x, y) = ln(1 + yx ).
x3 y+y2 x
9. f (x, y) = x+y
.

Exercise 7:
Let f be a function defined as follows:
 x3 +y3

 x2 +y2 if (x, y) , (0, 0)
h(x, y) = 

0
 if (x, y) = (0, 0)
1. Prove that the function f is continuous on R2 .
2. Prove that the function f is of class C 1 on R2 .
3. Show that the function f has first partial derivatives at (0, 0).
4. Is it differentiable on R2 ?
Exercise 8: a) Calculate the second partial derivatives of the functions: 1. f (x, y) = arctan( yx ).
2. g(x, y) = ln(x2 + y).
∂2 f ∂2 f ∂2 g ∂2 g
b) Verify that: ∂x∂y
(x, y) = ∂y∂x
(x, y) and ∂x∂y
(x, y) = ∂y∂x
(x, y).

110
(ESNN)S.Azoug 8.9. SERIES OF EXERCISES ON FUNCTIONS OF TWO VARIABLES
c) Let f be defined as follows:
x.y(x +y )
 2 2
 √ x2 +y2 if (x, y) , (0, 0)


f (x, y) = 

if (x, y) = (0, 0)

0

∂2 f ∂2 f
Show that: ∂x∂y
(x, y) , ∂y∂x
(x, y).
Exercise 9: Let f be a function defined on R2 by: f (x, y) = xy(x + y − 1).
∂f ∂f
• Calculate: , .
∂x ∂y

• Determine the critical points of f .

• Indicate their nature.

Exercise 10: In each of the following cases, determine an equation of the tangent plane at the
point (x0 , y0 ).

• f (x, y) = x2 + y2 at the point (x0 , y0 ) = (1, 1).

• f (x, y) = x2 + y2 at the point (x0 , y0 ) = (0, 0).


√ √
π π
• f (x, y) = sin(x.y) at the point (x0 , y0 ) = ( 2
, 3
).

111
(ESNN)S.Azoug CHAPTER 8. FUNCTIONS OF TWO VARIABLES
8.9.1 Correction of the series of exercises on functions of two variables
Exercise: 1.

Figure 8.4: Graphical representation of D1

Figure 8.5: Graphical representation of D2

Figure 8.6: Graphical representation of D3

Exercice: 2.

112
(ESNN)S.Azoug 8.9. SERIES OF EXERCISES ON FUNCTIONS OF TWO VARIABLES

Figure 8.7: Graphical representation of D4

Figure 8.8: Graphical representation of level curves C1

Figure 8.9: Graphical representation of level curves C2

Figure 8.10: Graphical representation of level curves C3

Exercice: 3.

113
(ESNN)S.Azoug CHAPTER 8. FUNCTIONS OF TWO VARIABLES

Figure 8.11: Graphical representation of level curves C4

Figure 8.12: Graphical representation of level curves C6

x3 y+y2 x
Figure 8.13: Graphical representation of the function: f (x, y) = x+y

Figure 8.14: Graphical representation of the function: f (x, y) = 1


x2 +y2
.

114
(ESNN)S.Azoug 8.9. SERIES OF EXERCISES ON FUNCTIONS OF TWO VARIABLES

Figure 8.15: Graphical representation of the domain of the definition : f (x, y) =


p
ln(xy)

Exercise: 4.
x2 + xy + y2 = 0.
p
Calculate the limits if they exist or show that they do not exist 1. lim
(x,y)−→(0,0)
2. lim 1
=∞
(x,y)−→(0,0) x +y
2 2

x.y
3. lim √ 0(C.Polaire 6 r)
(x,y)−→(0,0) x2 +y2
1
4. lim x−y (n0 existe pas)
(x,y)−→(1,1)
3
5. lim (x−1)y 2 +y2 0. (C.Polaire x − 1 = r cos(θ), y = r. sin(θ))
(x,y)−→(1,0)
6. lim (x + y) sin( x2 +y
1 1
2 )0.(| sin( x2 +y2 | 6 1)
(x,y)−→(0,0)
x2 +y2
7. lim 4 = ∞
(x,y)−→(0,0) x +y
4

2 = 0.(C.Polaire | f (r, θ)| 6


x+y 2
8. lim −→ 0.)
(x,y)−→(∞,∞) x +y
2 r
y. sin(x+y) 0
9. lim x2 −1
(n existe pas)
(x,y)−→(1,2)
10. lim x
xy−2y 0
2 +y2 −4x+4 (n existe pas = y(x−2)
(x−2)2 +y2
)
(x,y)−→(2,0)
Exercice: 5.
Examine the continuity of the following functions:
1.
x.y2



 si (x, y) , (0, 0)
f (x, y) =  x + y2
p 2



0 si (x, y) = (0, 0)

Ind. 0 6 | f (r, θ) − f (0, 0)| 6 r −→ 0 (d’ou la continuité en (0,0))


2.  x
y . sin( y ) si y , 0

 2
g(x, y) = 

0 siy = 0


Ind. 0 6 | f (x, y) − f (0, 0)| 6 y2 −→ 0 (d’ou la continuité en (0,0))


3.
x + y3
 3

si (x, y) , (0, 0)


h(x, y) =  x + y2

 2

0 si (x, y) = (0, 0)


Ind. 0 6 | f (x, y) − f (0, 0)| 6 |x + y| −→ 0 (d’ou la continuité en (0,0))

115
(ESNN)S.Azoug CHAPTER 8. FUNCTIONS OF TWO VARIABLES
4.
1 2

.x + y2 − 1 si x2 + y2 > 1.



2

k(x, y) = 

 1
− x2 sinon




2
Ind. 0 6 |K(x, y) + 2 x | −→ 0
1 2

Exercice: 6.
Représenter graphiquement le domaine de définition ainsi que les dérivées partielles premières:
1. f (x, y) = y5 − 3xy.
2. f (x, y) = x2 + 3xy2 − 6y5 .
3. f (x, y) = x. cos(e x.y ).
4. f (x, y) = yx .
5. f (x, y) = xy
6. f (x, y) = x2ln(x)
√+y −9
2

7. f (x, y) = x.y.
8. f (x, y) = ln(1 + yx ).
3 2
9. f (x, y) = x y+y
x+y
x
.
Exercice: 7.
Soit f une fonction définie par:

x + y3
 3

si (x, y) , (0, 0)


h(x, y) =  x + y2

 2

0 si (x, y) = (0, 0)


1.La fonction f est continue sur R2 car elle est le rapport de deux polynômes donc continue sur
R2 − (0, 0) .
En (0,0) car ind. 0 6 | f (x, y) − f (0, 0)| 6 |x + y| −→ 0 (d’ou la continuité en (0,0))
2.La fonction f est de classe C 1 sur R2 − (0, 0) car elle est le rapport de deux polynômes
En (0,0)?
∂f
• ∂x
(0, 0) = lim f (t,0)− f (0.0)
t
=1
(x,y)−→(0,0)

∂f
• ∂y
(0, 0) = lim f (0,t)− f (0.0)
t
= −1
(x,y)−→(0,0)

On déduit qu’elles existent et que f est de classe C 1 sur R2


4.Est-elle différentiable sur R2 ?
On écrit la formule suivante:
∂f ∂f
f (h, k) = f (0, 0) + (0, 0).h + (0, 0).k + k(h, k)k.ϕ(h, k)
∂x ∂y

∂f ∂f
• f (h, k) = f (0, 0) + ∂x
(0, 0).h + ∂y
(0, 0).k + k(h, k)k.ϕ(h, k)
(h−k)3

• h2 +k2
=h−k+ h2 + k2 .ϕ(h, k)

• ϕ(h, k) = √
−2.hk(h−k)
h2 +k2 .(h2 +k2 )

116
(ESNN)S.Azoug 8.9. SERIES OF EXERCISES ON FUNCTIONS OF TWO VARIABLES
• On montrera que ϕ(h, k) −→ (, 0)(methode des chemins)

• Calculer: ϕ(h, 0) , ϕ(h, 2h) avec h −→ 0. on déduit que f n’est pas différentiable.

Exercice: 8.
a) Calculer les derivées partielles secondes des fonctions :
1. ∂ ∂y∂x = ∂ ∂x∂y
2 f (x,y) 2 f (x,y) 2 −y2
= (xx2 +y 2 )2

2. ∂ ∂y∂x = ∂ ∂x∂y
2 2
g(x,y) g(x,y)
= x−2x
2 +y2

c) Soit f définie par:


x.y(x2 + y2 )



 si (x, y) , (0, 0)
f (x, y) =  +
 p 2

2
 x y

0 si (x, y) = (0, 0)

∂ f 2 ∂ f 2
On trouve: ∂x∂y (0, 0) = 0, ∂y∂x (0, 0) = ∞
Exercice: 9.
Soit f une fonction définie sur R2 par:

f (x, y) = xy(x + y − 1)
∂f ∂f
• Calculer: ∂x
= 2xy + y2 − y, ∂y
= x2 + 2xy − x

• Les points critiques de f sont (0,0),(1,0),(0,1),(1/3,1/3).

• Leurs natures.
Points S r t S 2 − rt nature
(0,0) -1 0 0 1 rien conclure
(1,0) 1 0 2 1 rien conclure
(0,1) 1 2 0 1 point selle
(1/3,1/3) 1/3 2/3 2/3 -3/9 minimum local

Exercice: 10.
L’ equation du plan tangent au point (x0 , y0 ).

• z = 2x + 2y − 2 au point (x0 , y0 ) = (1, 1)

• z = 0 au point (x0 , y0 ) = (0, 0)

√ √ √ √
π 3 π 3 π π
• z = 1/2 − 6
+ 6
(x + y) au point (x0 , y0 ) = ( 2
, 3
)

117
(ESNN)S.Azoug CHAPTER 8. FUNCTIONS OF TWO VARIABLES

118
Chapter 9
Courbes Planes

119
(ESNN)S.Azoug CHAPTER 9. COURBES PLANES

120
Chapter 10
Exercise Solutions

10.1 Solutions to exercises on real numbers


Exercise : 01. (Knowledge Check)** Determine whether the following statements are true or
false:
Let a, b, and k be real numbers such that 0 ≤ a ≤ b. Then:
1. a · k ≤ b · k *(True if k ≥ 0)
2. a · k ≤ a · k2 *(False, e.g., for a = 0.1 and k = 0.1)
3. a · k ≤ a · k3 *(False, e.g., for a = 0.1 and k = 0.1)
4. ak ≤ bk *(True if k > 0)
5. ka4 ≤ kb4 *(True if k , 0)

Exercise : 02.
Determine whether the following statements are true or false:
Let a, b, c, and k be real numbers such that 0 ≤ a ≤ b ≤ c. Then:
1. a · k ≤ b · k ≤ c · k *(True if k ≥ 0)
2. a · k ≤ b · k2 ≤ c · k3 *(False, e.g., for a = 0.1, b = 0.2, c = 0.3, and k = 0.1)
3. a · k2 ≤ b · k2 ≤ c · k2 *(True)
4. ak ≤ bk ≤ kc *(True if k > 0)
5. ka2 ≤ kb2 ≤ kc2 *(True if k , 0)
Exercise : 03.(Applications)
Show that for all a, b, c in R:

a b a2 b2
1. + > 2 *(Write: (a − b)2 > 0 ⇒ a.b + a.b > 2 if a.b > 0.)
b a
2ab √ a+b √ √
2. 6 ab 6 *(Write: ( a − b)2 > 0.)*
a+b 2
3. Deduce: ab + bc + ac 6 a2 + b2 + c2 *(Write: 2ab 6 a2 + b2 , 2ac 6 a2 + c2 , 2cb 6 c2 + b2 ,
then add them up.)*

Exercise : 04.(Applications)
Let x, y be real numbers such that:

121
(ESNN)S.Azoug CHAPTER 10. EXERCISE SOLUTIONS

1 6 x 6 2, and 2 6 y 6 3.

Provide bounds for A and B:


!2 s
2x + 1 2x2 + 2x + 1
A= , B=
3y − 1 x3 + 3y − 1

1. *(Write: 3 6 2x + 1 6 5; 1
8
6 1
3y−1
6 15 .)*
2x+1 2
 
9
2. *(Using multiplication and squaring: 64
6 3y−1
6 1.)*

Similarly:
1. *(Write: 5 6 2x2 + 2x + 1 6 13; 1
16
6 1
x3 +3y−1
6 1 .)*
√ q 6
q
5 2 +2x+1
2. *(Using multiplication and squaring: 4
6 2xx3 +3y−1
6 136 .)
Exercise : 05.(Applications) Let x, y be real numbers. Solve the following systems:

1. 
2x − 5 < 4x − 2



6x + 1
>1




2
*(Solution: x > −1
6
)*
2. 
2x + 8y = −6


 x − y2 = 1

*(Solution: O5 = 5, O6 = 6.| − 2| = 12)*

**The Powers**

Exercise : 06. 06(Knowledge Check) Determine if the following statements are true or false:

Let a, b, c, and k be real numbers

1. 2 x + 2 x = 22x *(False; take x = 0.)*


2. (an + bn )2 = an + bn + 2an bn *(False; take a = 0, b = n = 2.)*

Exercise : 07.(Applications)
*(Solution: O3 = 1.)*
Exercise : 08.(Knowledge Check)
Let x, y be real numbers

122
(ESNN)S.Azoug 10.1. SOLUTIONS TO EXERCISES ON REAL NUMBERS
1. (x + y) = x + 4x y + 6x y + 4xy3 + y4
4 4 3 2 2

2. a = 25 15
20
3. O1 = 8.
**Roots**
Exercise : 09. (Knowledge Check)
Determine if the following statements are true or false for real numbers a and b:


√a = a. *(False if a < 0.)*
1. 2

2. (√ a)2√= a. *(True.)*

3. √a · b =
√ √ab. *(True.)*
4. √ab = a · b. *(False; a = −2, b = −3.)*
√a + b =√a + b.√*(False; a = 1, b = −1.)*
5. 2 2

6. pa + b = a + b. *(False; a = 1, b = 1.)*
7. (a + b)2 = a + b. *(False if a + b < 0.)*

Exercise : 10.(Applications)
Show that for all positive real numbers a, b, and c:

√ √ √ √ √
1. √a + b 6 a + b. *(Write: ( a + b)2 > 0.)*
2. ab + bc + ac 6 a + b + c. *(Write: (a2 + b2 + c2 )2 > 0.)*

Exercise : 11.(Applications)
Solve in R:

√ √
1. 2x − 1 6 4x + 1. *(Solution: S = [ 12 , +∞[.)*
√ √
2. x − 2 6 3x + 1. *(Solution: S = [ −1 , +2[∪[2, 7+ 2 37 ].)*
√ 3
3. x − 4 4x − 19 > 1.
**Absolute Value**

Exercise : 12.(Knowledge Check)


Indicate if the following statements are true or false for real numbers a and b:

1. |a2 b| = b · a2 *(False if b < 0.)*


2. |a2 | = a2 *(True.)*
3. |a + b| = |a| + |b| *(False; a = 2, b = −2.)*
4. |a − b| 6 |a| + |b| *(True, triangle inequality.)*

Exercise : 13.(Applications)
Solve in R:

1. |2x − 1| = x + 1, *(Solution: O2 = 5 · 2 + 1 = 11.)*


2. |2x − 1| 6 3

123
(ESNN)S.Azoug CHAPTER 10. EXERCISE SOLUTIONS
3. |2x − 2| + |x + 1| = 3
4. |2x − 2| − |x + 1| 6 3

** Upper bound, Lower bound, Sup, Inf**

Exercise : 14.
Find, if they exist, the Max, Min, Sup, and Inf of the following sets:

1. A = {−1, ) (O4 = S up(A))


( 7, 8, −3, 13},
1
2. B = 1 − , n ∈ N∗
( n )
1 n
3. C = + ,n ∈ N
2 1+n

Exercise : 15.(Applications)
Let A and B be two non-empty subsets of R such that A ∩ B , ∅.

If A ⊂ B, then sup(A) 6 sup(B) and in f (A) > in f (B).

Exercise :(Code to Find?) Find the next word:

Pro f : O1 .O2 O3 O4 O5 O6

**Important Note: The first person to provide the code will receive +.. points on the partici-
pation.**

124
(ESNN)S.Azoug 10.1. SOLUTIONS TO EXERCISES ON REAL NUMBERS
**Solutions to Exercises on Number Sequences**
Exercise : 01.(*Recurrence*) Prove by induction the following relations:
n(n + 1)
1. nk=1 k2 =
P
2
2. 1 + 3 + 5 + .... + (2n − 1) = n2
1 n
3. nk=1 =
P
(2k − 1)(2k + 1) 2n + 1

Exercise : 02.(*Sum and Product*)


Express using the symbols Π and up to order n for the following relations:
P
1
1. 1·2 + 2·3
1
+ 3·41
...
2. 1·2·21 + 2·3·22 + 3·4·2
1 2 3
3 ...

3. sin(2x) + sin(3x2 ) + sin(5x


1 1 1
3 ) ...
e1 e2 e3
4. 1
· 2
· 3
...

Exercise : 03.(*Knowledge Check*)


1. Is the sum of two divergent sequences necessarily divergent?
2. Can the product of two divergent sequences result in a convergent sequence?
3. If (Un ) is a decreasing positive sequence, does it necessarily imply lim Un = 0?

Exercise : 04. (*Nature of a Sequence*)


Study the nature of the following√sequences: (a, α are real numbers)
√ n−1
1. a)Un = n − n2 − n, b)Un = √
n+1
nn
2. c)Un = , d)Un = (1 + na )n
n!
cos(nα)
3. e)Un = , f )Un = 2n − 3n
n
Exercise : 05. (*Bounding*)
Consider the numerical sequence (Un ) defined by Un = nk=0 n+1√k .
P
Using a suitable bounding of Un , determine the limit of this sequence.

*Exercise : 06.(*Convergence Theorem*)


Un2
Consider the numerical sequence (Un ) defined by U0 = 1 and Un+1 = + 1.
4
1. Prove by induction that (Un ) is increasing and bounded by 2.
2. Calculate its limit.

Exercise : 07.
Let θ be a real number in the interval [0; π2 ]. Consider the sequence (Un ) defined by:

U0 = 2 cos(θ) and Un+1 = 2 + Un .
1. Calculate U1 and U2 in terms of θ.
2. Show that Un = 2 cos( 2θn ), and calculate its limit.

Exercise : 08.

125
(ESNN)S.Azoug CHAPTER 10. EXERCISE SOLUTIONS
Show that the two sequences (Un ) and (Vn ) defined by:
Un = nk=3 1+k
1
2 and Vn = U n + n − 2n2 are adjacent.
1 1
P

Exercise : 09.
Consider the sequence (Un ) defined by:
U0 = 1, Un+1 = Un e−Un .
1. Show that this sequence is positive, decreasing, and converges. Calculate its limit.
2. Let S n = nk=1 Uk . Prove that for all n, Un+1 = e−S n .
P
3. Deduce the limit of S n .

Exercise : 10.(*Fixed Point*)


*Note: See reminders.*
Using the fixed-point theorem, | f 0 (x)| = 12 cos(x) 6 12 , which means f (x) is a contraction.
Also, I = [0, 1] ⊆ [0, 1].
This function has a fixed point, and the sequence converges to this fixed point.
Exercise : 11.(*Equation of Leonardo de Pisa (1225)*) Consider the function f (x) = x3 +2x2 +
10x − 20.
1. Show that the equation f (x) = 0 has a real root on the interval ]1; 2[.
20
2. Prove that the equation f (x) = 0 can be written as F(x) = x, where F(x) = 2 .
x + 2x + 10
3. Show that |F 0 (x)| < 1 for x in ]1; 2[.
4. Deduce that the sequence Un = F(Un−1 ) is convergent. What is its limit?
5. By taking U0 = 1, calculate U1 , U2 , ..., U24 using a computer.

**Homework (.. pts)**


1.) **Leonardo de Pisa** found 1.368808107, now it’s your turn to find U30 .
2.) Provide the fixed-point theorem.

126
(ESNN)S.Azoug 10.2. SOLUTIONS TO EXERCISES ON FUNCTIONS
10.2 Solutions to exercises on functions
Exercise: 01. (Domain of Definition)
The domainrof definition for each of the following functions:
1−x
1. f1 (x) =
4 − x2

D1 =] − 2, 1]∪]2, +∞[

2. f2 (x) = ln(x2 − 3x + 2)

D2 =] − ∞, 1[∪]2, +∞[

ln(x2 − 3x + 2)
3. f3 (x) =
|x2 − 3|
√ √
D3 =] − ∞, − 3[∪] − 3, 1[∪]2, +∞[

4. 
1
 if (x > 0)
f4 (x) = 

 2−x
 x2 − 3x + 2 if (x < 0)

D4 =] − ∞, 2[∪]2, +∞[

Exercise: 02. (Hyperbolic and Trigonometric Functions)


Determine the domain of definition for each of the following functions:
1. f2 (x) = 5 sinh(x) − 3 cosh(x)

D4 =] − ∞, +∞[

2. f2 (x) = sinh(x)
sinh(2x)

D4 =] − ∞, 0[∪]0, +∞[

3. f2 (x) = 2 tanh(2x)

D4 =] − ∞, +∞[

4. f2 (x) = 3 cos3 (x)

D4 =] − ∞, +∞[

127
(ESNN)S.Azoug CHAPTER 10. EXERCISE SOLUTIONS

5. f2 (x) = tan(x)

π
D4 =] − ∞, +∞[−{ + 2kπ}
2

Exercice : 03.

Calculate the following limits:



x3 −2x2 +3 e x+1
A. lim x−→0+ x2
x+2
ln(x)
= −∞; lim x−→+∞ x. ln(x)
= +∞; lim x−→+∞ x+2
= ∞; lim x−→0+ ln(3x+1)
2x
= 3
2

x x −1 x 3
+4
B. lim x−→0+ ln(x+1)
= R.H (−∞); lim x−→−∞ 2
x+1
. ln( ) = 0(∼ ln(−x)
x
); lim x−→0+ (1+ x)ln(x) =
x+1
1 − x2
1(∼ e x. ln(x) ); lim x−→+∞ ( xx2 +5
3 ln(x)

+2
) x2 +1 = 1(∼ e x )

√ √
q √ q √
x. ln(x2 +1) n −1
C. lim x−→+∞ 1+e x−3
= 0(∼ x. ln(x)
ex
); lim x−→1 xx−1 = n; lim x−→+∞ ( x+ x2 + 1− x+ x2 − 1) =
0(con jugue)

Exercise : 04.(Limites )
x −e2
Calculate the limits of the following functions at the point x0. 1.lim sin(ln(x)−1)
x−e
..x0 = e; lim xe2 +x−6 ..x0 =
2 √ √
√ ..x0 = 4; lim
2x+1−3
2.lim √ x−2− x. sin( √1x ) ..x0 = 0
2
5 2x 2
3..lim xx3 −1
−1
..x0 = 1; lim ee3x −e
−e
2 ..x0 = +∞
ln(e3x −1)
4.lim sin(2x) ..x0 = 0; lim x−e ..x0 = +∞
sin(ax)
√ √
5.lim x. ln( √ x2 ) ..x0 = +∞ ; lim 1+x− 1−x
..x0 = 0
x +1 x
6.lim 1−cos(x)
x2
..x0 = 0 ; lim( x+1
x−e
) x ..x0 = +∞

Exercise : 05.
Determine the domain of definition of the following functions and their limits at the boundaries
:

• f (x) = ln(ln(x))
ex − 1
• f (x) =
ex + 1
1
ex
• f (x) =
x
• f (x) = ln(| x |)

128
(ESNN)S.Azoug 10.2. SOLUTIONS TO EXERCISES ON FUNCTIONS
Exercice : 06.
Calculate the limits at 0 of the following functions :

sin(2x)
• lim x→0 =2
x
sin(2x)
• lim x→0 = 2
3
sin(3x)
tan(ax)
• lim x→0 = a
b
sin(bx)
x − sin(ax)
• lim x→0
x + sin(3x) = 1−a
4

x2
• lim x→0 =0
sin(πx)
• lim x→0 x. sin( 1x ) = 0

Exercise : 07

Calculate f ◦ h and h ◦ f for the following functions and draw conclusions.

• f (x) = x2 , h(x) = ln(x)

f oh(x) = ln2 (x); ho f (x) = ln(x2 )

There is no equality.

• f (x) = x2 , h(x) = e x
2
f oh(x) = e2x ; ho f (x) = e x
There is no equality.

• f (x) = ln(x), h(x) = e x

f oh(x) = x, x ∈ R ; ho f (x) = x, x > 0.

There is no equality with condition.



• f (x) = x, h(x) = x2
f oh(x) = |x|; ho f (x) = x, x > 0.
There is no equality with condition.

Exercice : 08.
The parity of functions :

ex − 1
• f (x) = f is odd
ex + 1

129
(ESNN)S.Azoug CHAPTER 10. EXERCISE SOLUTIONS
• f (x) = 1 + x2 + cos(3x) f is even

• f (x) = x + x3 + tan(2x) f is odd

x2 + sin2 (x)
• f (x) = f is even
| x | +6

x2
• f (x) = f is odd
sin(πx)

• f (x) = x. sin( 1x ) f is even

Exercice : 09.

Determine the numbers a and b for which f is continuous on R.

x2 − 3x x<2
(
f (x) =
2x + bx x>2
eax x<1




g(x) =  x=1

2



 x2 + b2

x>1

Exercice : 10.(Continuité)

1 Study the continuity of the function f at the point x0 :

f (x) =| x − 1 | x0 = 1
f is continuous because : lim x→1 | x − 1 | = 0 = f (1)

2 Study the continuity of the function f at the point x0 = 0 :


1

 x . cos( ) si(x , 0)

 2
f (x) = 

x
0 si(x = 0)


3 For what value of a is f continuous at 0? ?


√ √

 1 + x − 1−x
 si(x , 0)
f (x) = 


 x
3a2 − 27 si(x = 0)


130
(ESNN)S.Azoug 10.2. SOLUTIONS TO EXERCISES ON FUNCTIONS
4 For which values of α and β is f continuous at 2?

x + α.x − β si(x 6 2)
( 2
f (x) =
2x + β si(x > 2)

Exercise : 11.((Continuity Extension) Can the following functions be extended to 0?


1. f5 (x) = x.ln(|x|)Oui
2. f5 (x) = x − |x|x Non
3. f5 (x) = sin(x). sin( 1x )Oui
Exercise : 12.(Inverse) Let f be the function defined by :

x si(x < 1)





f (x) = 
 2

x si(1 6 x 6 4)

 √
8 x si(x > 4)

1. Plot the graph of f . 2. Is f continuous over R? Yes. 3. Show that f has an inverse function
f −1 for which the expression is given by: (expression of f −1 ):
√


 x si(x < 1)

 x2 si(1 6 x 6 16)


f −1 (x) = 

x2



si(x > 16)




64

Exercise : 13.(Inverse)
Let the function f be defined by :
x
f (x) =
1 + |x|
1. Show that f establishes a bijection from R onto f (R).
2. Determine f (R) and f −1 .
Exercise 14.
Consider a function f from [a, b] to R that is continuous and satisfies f (a) = f (b). Show that
the function g defined by: !
b−a
g(t) = f t + − f (t)
2
vanishes at at least one point c in [a, a+b
2
].
!2
a+b b+a
! !
g(a) · g =− f − f (a) < 0
2 2
Using the Intermediate Value Theorem and the continuity of g.

Exercise: 15. (Differentiability)

Calculate the derivatives of the following functions :

131
(ESNN)S.Azoug CHAPTER 10. EXERCISE SOLUTIONS
√3 3x + 1
2
A. ( x2 + x + 1)0 =
3(x2 + x + 1) 3
2

(x x√)0 = x x .(ln(x) +
1) √
(e x+sin(x) )0 = ( 2 √1 x
+ cos(x))e x+sin(x)
3
( ln(3x+1) )0 = − ln(3x+1)
2x
2x(3x + 1) 2x2

B. (a x − 1)0 = ln(a).a x ;
x3 + 4 0 3x2 2x
(ln( )) = + ;
1−x 2 4+x 3 1 − x2
ln(x + 1) ln(x)
((1 + x)ln(x) )0 = ( + )(1 + x)ln(x) ;
x x+1
3x3 2x2 x3 + 5 x3 +5 x
!
x3 +5 x 0
(( x2 +2 ) ) = − + ln( ) ( 2 )
5 + x3 2 + x2 2 + x2 x +2

C. (tanh(x2 + 1))0 = 2x(1 − tanh2 (x2 + 1));


(sh2 (x))0 = 2ch(x).sh(x);
(arctan(3x))0 = 3(1 + arctan2 (3x));
1
(arcsin(2x + 5))0 = p
−(x2 + 5x + 6)

Exercise : 16.

Study the differentiability of the following functions on their domains of definition and then
provide the expressions for their derivatives.
 x
 1+ | x | x < 2


f (x) = 


2x x > 2


 x

 x,0
+
1
g(x) = 

1 e
 x

0 x=0


Exercise : 17.

On considère la fonction f définie par :

sin(x2 )


 x,0
f (x) = 


 x
0 x=0

1. Show that f is differentiable over R.

B. Calculate the derivative of f .

132
(ESNN)S.Azoug 10.2. SOLUTIONS TO EXERCISES ON FUNCTIONS
1
C. Show that f is of class C over its domain of definition.

Exercise : 18.

Calculation of the nth derivatives :

1. f (x) = (x2 − x + 2)e x

f (n) (x) = (x2 + (2n − 1)x + 3n + 2)e x , n − impaire


f (n) (x) = (x2 + (2n − 1)x + 2n + 6)e x , n − paire

2. f (x) = cos2 (x) sin(x)

f (n) (x) = an . cos(x) sin2 (x) + bn−1 cos3 (x), n − impaire


f (n+1) (x) = bn cos2 (x) sin(x) − an sin3 (x).
bn = 3|an | + 1, an+1 = 9|an | + 2, b0 = 1, a1 = −2.

1
3. f (x) =
x2 −1

Exercice : 19.

The relation is : ( Pn and Pn+1 )


1
Pn+1 = P0n .(1 + x2 ) − 2.(n + ).Pn
2

Exercice 20.(T.V.I)

A. e use I.V.T and ( Newton’s binomial)


fn (1). fn (1 + n1 ) = −(1 + Reste(n)) < 0. ⇒ fn (xn ) = 0.

B. fn+1 (xn ) = xnn (xn − 1) > 0, car xn > 1.

C. fn+1 (the function is increasing) and as fn+1 (xn+1 ) = 0 < fn+1 (xn ) ⇒ xn+1 < xn ( the
sequence (xn ) is decreasing and lower bounded so convergent to l=1.

Exercise : 21.

133
(ESNN)S.Azoug CHAPTER 10. EXERCISE SOLUTIONS
A. We use F.A.T
| sin(x) − sin(y)| = |(x − y) cos(c)|
sachant que : | cos(c)| 6 1
| sin(x) − sin(y)| 6 |x − y|

B. We use F.A.T x → ln(x + 1)


1
ln(x + 1) − ln(0 + 1) = (x − 0).
1+c

Exercise : 22.

Soit f : [a, b ] → R une application continue, on suppose f [est dérivable sur ]a, b [ et que
∀x ∈ [a, b], f (x) > 0. :

• We use F.A.T x → ln( f (x))

f 0 (c)
ln( f (b)) − ln( f (a)) = (b − a).
f (c)

f 0 (c)
f (a) (b−a)
=e f (c)
f (b)

Exercise : 23.(Els)

Calculate the following limit expansions :

A.
sinh(2x) = 2x + o(x2 )

B.
x2
sinh(x) cosh(x) − cosh(x) = −1 + x − + o(x2 )
2

134
(ESNN)S.Azoug 10.2. SOLUTIONS TO EXERCISES ON FUNCTIONS
C.
x3
arctan(x) = x − + o(x3 )
3

E.
x3
argsh(x) = x − + o(x3 )
6

F.
x2 + 1 3x2
ln( ) = ln(x2 + 1) − ln(x + 1) = −x + + o(x2 )
x+1 2

G. 2
ln(x + 1) x − 2 + o(x2 )
x
x2
= = x − − x3 + o(x3 )
(x2 + 1) x2 + 1 2

I.
f (x) = x3 − 3x2 + x + 2 + o(x2 )

Exercise : 24.

Calculer les limites en 0 :

A. 3
x − arc sin(x) x − (x + x6 ) −1
lim = lim =
sin3 (x) x−→0 ((x + x )3
3
x−→0 6
6

B. 2
ln(x + 1) (x − x2 )
lim = lim =1
x−→0 tan(x) x−→0 x

C. 3 4
1 1 x2 − (x − x6 )2 x
1
lim − = lim = lim 3
=
x−→0 sin2 (x) x2 x−→0 x2 .(x − x3 )2 x−→0 x4 .(1 − x2
) 3
6 3

135
(ESNN)S.Azoug CHAPTER 10. EXERCISE SOLUTIONS
10.3 Solutions to exercises on integrals

Exercise : 1.(Powers)
Calcul
R des primitives :
1. (5.x4 − 3.x2 + 6.x − 7)dx = x5 − x3 + 3x2 − 7x + c
√ √ 1 √ √ √
2. (5. x3 − 3. x.x2 + 6. √ )dx = 2 x5 − 67 x7 + 12 x + c
R
x
R 1 x2 3 √
3. ( 2 − 3. √ + 6. √x 3 )dx = −5 x
+ 65 x5 + c
x x x
Exercise : 2.(trigonometric)
Calculation of antiderivatives :
1.R (5.sin(x) − 3.cos(x) − 7)dx = −5 cos(x) − 3 sin(x) − 7x + c
R

2.R (2.sin(3x) − 3.cos(2x))dx = − 23 cos(3x) − 32 sin(2x) + c


3.R (2.sin2 (x))dx = − 12 sin(2x) + x + c
4.R (3.cos3 (x))dx =
5. (tan(x))dx = − ln(| cos(x)|) + c
Exercise : 3.(change of variable)
Calculation
R sin(ln(x)) of antiderivatives :
1. )dx = − cos(ln(x)) + c
R 1+e √x x √ √
2. √ x .dx)dx = 2 x + 2e x + c
R ex
x )dx = ln(|1 + e |) + c
x
3. 1+e
q
√1 )dx = −2 ln( 1x ) + c
R
4.
x. ln( 1x )
x 2 x 2
5. e x ln(1 + e x ))dx = ( (1+e2 ) − (1 + e x )) ln(1 + e x ) − (1+e4 ) + (1 + e x ) + c
R

avec les changements (t = ln(x), t = x, t = e x , t = ln(1/x), t = 1 + e x )
Exercise : 4.(Formulas)
Calculation of antiderivatives :
(x + 3)8
1. ((3 + x)7 )dx = +c
R
8
2. 41 (4.(4x + 3)1/3 )dx = 16 (4x + 3)4/3 + c
R 3
x
12
3. (22x .3 x )dx = +c
R
ln(12)
4. (x.e x )dx = 21 e x + c
R 2 2

Exercise : 5.
Calculation of antiderivatives :
2
1.R (x. ln(x))dx = x2 (ln(|x|) − 12 ) + c
R

2.R (arctan(x))dx = x2 arctan(x) − 12 ln(x2 + 1) + c


3.R (x. cos(x))dx = x. sin(x) + cos(x) + c
4.R (ln2 (x))dx = x ln2 (x) − 2x ln(x) + 2x + c
x
5. (cos(x).e x )dx = e2 (sin(x) + cos(x)) + c
Exercise : 6.(Fractions)
Calculation
R 5x+3 of antiderivatives :
1.R ( x2 −3x+2 )dx = R (−10 − x−1 − x−2 )dx = −10x − 8 ln(|x − 1|) − 13 ln(|x − 2|) + c
R 8 13

2. ( x2 +4x+5 )dx = ( (x+2)2 +1 )dx = arctan(x + 2) + c


1 1

136
(ESNN)S.Azoug 10.3. SOLUTIONS TO EXERCISES ON INTEGRALS
3. ( x3 +1 )dx = ( 3(x+1) + 6(x2 −x+1) + 2(x2 −x+1) )dx = 13 ln(|x+1|)− 16 |x2 − x+1|+ √13 arctan( √2x3 − √13 )+c
R 1 R 1 −(2x−1) 1

)dx =
R
4. ( 2x−3
x3 +x
Exercise : 7.
For every integer n, let’s define:
Z 1 √
In = xn · 1 − x dx
0

1. By calculating In−1 − In , deduce a relation between In − In−1 .


2. Calculate I0 .
3. Deduce In .

137
(ESNN)S.Azoug CHAPTER 10. EXERCISE SOLUTIONS
10.3.1 Fixed Point Theorem
Let I be a closed interval in R, f : I −→ I a contraction mapping (i.e., there exists k ∈]0, 1[
such that for all x, y in I: | f (x) − f (y)| ≤ k|x − y|. Then f has a unique fixed point l. Moreover,
any sequence defined by u0 ∈ I, un+1 = f (un ), converges to this unique fixed point, and we have
the following estimate:
|un − l| ≤ kn |u0 − l|

10.3.2 Result on Fixed Point


Let f : E −→ E be a continuous function, and (un ) be a recurrent sequence defined by u0 ∈ E
and un+1 = f (un ). If (un ) converges, it must converge to a fixed point of f .

138
Bibliography

[1] K. Allab, Éléments d’analyse, Fonctions d’une variable réelle, 1ere et 2eme annee
d’universite, Ecoles scientifiques -OPU- 1984.

[2] . Rivaud (1982). Séries et Équations Différentielles (Cours et Exercices).

[3] E. Azoulay, Problemes corrigées de mathématiques - 2ed. Paris : Dunod,2002.

[4] V. Smirnov (1981). Cours de Mathématiques Supérieures.

[5] N. Piscounov, Calcul differentiel et intégral Tome 1, 9eme edition, Edition Mir, Moscou,
1980.

[6] . Demidovitch (1977). Recueil d’Exercices et de Problèmes d’Analyse mathématiques.

[7] L. Chambadal, Exercices et problèmes résolus d’analyse : mathématiques speciales. Bor-


das, 1973.

[8] A. Hitta, Cours d’algebre et exercices corrigees. O.P.U., 1994.

[9] N. Piscounov, Calcul differentiel et integral Tome 2, 7 edition, Edition Mir, Moscou, 1980.

[10] C. Baba-Hamed, K. Benhabib, Analyse 2, Rappel de cours et exercices avec solutions.


O.P.U., 2012.

139

You might also like